Categories
Cambridge Exam Questions

Cambridge. Intercollegiate and preliminary examinations in economics, 1931-33

 

The following examinations were included in a publication of the Cambridge University Economics Tripos for 1931-33. Appended to the publication of the 1921-1926 Cambridge Economics Tripos Papers are the “Papers set in the qualifying examinations 1925 & 1926”. So the following three exams most likely served as an early hurdle to clear before being admitted to the Economics Tripos.

 

Intercollegiate Examination of Economics (June 1931)

Principles of Economics
Subjects for an Essay
Trade and Finance
Industry and Labour

Intercollegiate Examination of Economics (June 1932)

Economic Theory
Currency and Banking
Industry and Trade
Labour

Preliminary Examination in Economics (June 1933)

Principles of Economics
Industry, Labour and Money. Paper I
Industry, Labour and Money. Paper II
Modern Economic History

_________________________

 

INTERCOLLEGIATE EXAMINATION IN ECONOMICS.

Monday, June 8, 1931. 9—12.
PRINCIPLES OF ECONOMICS.

  1. What is meant by a perfect market? To what extent must fundamental economic theory be modified in view of the fact that in practice markets are not perfect?
  2. “The representative firm was devised to meet the difficulties occurring in the analysis of supply when there is a disparity of efficiency as between different producers.” Comment.
  3. Are there any reasons, for purposes of economic analysis, for drawing a distinction between the royalties received by owners of mining properties and the interest received by debenture holders in mining companies?
  4. “Increasing returns maybe due to external economies or to internal economies.” What are external and internal economies, and under what conditions, and in what sense, can increasing returns be said to be due to each?
  5. “If we were content to dispense with further industrial progress we should no longer have to pay tribute in the form of profits to business men.” Discuss.
  6. What is likely to be the effect on the price and output of agricultural produce of levying a land tax from which agricultural land is exempt?
  7. In what circumstances is a rise in the general level of wages likely to be (a) compatible, (b) incompatible with full employment for labour?
  8. Consider carefully under what conditions the control of an industry by a monopolist is likely to be in the social interest.
  9. “The Theory of Distribution is but a particular application of the Theory of Value.” Elucidate this statement.
  10. How far do economic and social considerations justify the right of private bequest?

 

Monday, June 8, 1931.  1. 30—4. 30.
SUBJECTS FOR AN ESSAY.

Write an essay on one of the following subjects:

  1. “For our great-grandchildren the problem will be how to live rather than how to keep alive.”
  2. The Electrification of the Railways.
  3. “Democracy has had its chance and has failed: let us pass on.”
  4. The Control of the World’s Wheat.
  5. “The Five Years Plan.”
  6. The Economics of Advertising.
  7. “La République n’a pas besoin de Savants.”

 

Tuesday, June 9, 1931. 9—12.
TRADE AND FINANCE.

  1. “Ten years of industrial depression measures the cost to this country of the Return to Gold.” Discuss.
  2. Would you agree that the world price level of the next twenty years is at the discretion of the Central Banks in the chief industrial countries? Why, or why not?
  3. What bearing has the proportion of their resources that people choose to hold in the form of money upon the value of money?
  4. In October 1929 the market rate of discount was about 6¼ % while the yield of fixed-interest stocks was about 4¾ % in March 1931 the former was about 2½ % and the latter about 4½ %. How would you account for these facts?
  5. “To-day, as in the middle of the last century, Free Trade is the best policy for this country: the reasons may be different, the conclusion is the same.” Investigate this argument.
  6. What are the relative advantages or disadvantages for this country of a fixed fiduciary issue as compared with a proportional reserve system of note issue?
  7. State clearly the differences in organisation between the money markets of London and New York, and indicate how these differences affect the nature and extent of the influence of the Bank of England and the Federal Reserve Bank of New York in the two markets respectively.
  8. With what truth is it alleged that whereas before the war the sources of disturbance to the Foreign Exchanges automatically set in motion corrective forces, to-day this no longer tends to be so?
  9. How do you account for the differences in the levels of wages normally prevailing between different countries?
  10. What criteria should a Government hold in view in framing a system of taxes?

 

Tuesday, June 9, 1931.  1. 30—4. 30.
INDUSTRY AND LABOUR.

  1. “It was once regarded as the duty of the State to curb the power of monopoly, but it now does everything possible to encourage it.” Discuss the truth of this statement and the desirability of the change to which it alludes.
  2. Examine the effect that would be produced on the wages of skilled workers by the introduction of a national minimum weekly wage.
  3. Give a reasoned account of what you imagine is likely to be the position of the British cotton industry in ten years time.
  4. Describe the British Trade Boards system. What conditions are necessary for the establishment of a Trade Board in a particular industry and what conditions do you think ought to be necessary?
  5. Consider how far the existence of surplus capacity in an industry can be regarded as socially undesirable.
  6. “Since the war there has been a strong positive correlation between rates of real wages and unemployment.” Explain. Can any moral be drawn?
  7. What are the factors that determine the size of firms? Illustrate your answer by reference to actual industries.
  8. Discuss in the light of the experience of other countries the possibility of improving the efficiency of British farming on the side of (a) production, (b) marketing.
  9. How far and for what reasons is labour in this country immobile at the present time? What benefits, if any, would you expect from increased mobility?
  10. Is it possible for technical improvements in production to be adopted faster than is in the interests of the working classes? Illustrate your answer by reference to the conditions of to-day.

 

Monday, June 6, 1932. 9—12.
ECONOMIC THEORY.

  1. Explain the difficulties involved in measuring the total utility derived from any commodity in terms of money.
  2. “Under private enterprise the consumer is king.” How far do the teachings of modern psychology cast doubt upon the truth of this assertion?
  3. Give examples of the way in which the development of economic theory has been moulded by the course of historical events.
  4. Explain the forces which determine the value of a commodity “in the short period,” making plain what you mean by a “short period.”
  5. Is the existence of monopoly (a) a necessary, (b) a sufficient condition for the existence of a system of differential prices?
  6. “Since rents are merely transfer payments from one member of the community to another, it is impossible for an industry to be conducted under conditions of increasing cost from the standpoint of the community as a whole.” Discuss.
  7. Is there any reason, from the standpoint of the public interest, why the freights and fares on the English railways should be maintained at a level sufficient to yield a normal rate of interest on the capital originally invested in the railways?
  8. How far can the doctrine that each factor of production tends to be rewarded in accordance with its marginal productivity be applied to the factor risk-taking?
  9. “Since cuts in wage-rates destroy purchasing-power, they are bound to prolong and aggravate a trade depression.” Comment.
  10. Is the present pace of agricultural and industrial improvement likely in your opinion to make the problem of chronic unemployment greater in the 20th century than in the 19th?

 

Monday, June 6, 1932. 1. 30—4. 30.
CURRENCY AND BANKING.

  1. “The Purchasing Power of Money is a phrase which admits, not of one, but of many meanings, some more useful than others.” Elucidate this statement.
  2. Explain briefly the points at issue between the Currency and Banking Schools in the eighteen-forties. How far was the doctrine of the victorious school justified by the events?
  3. What support does history afford to the contention that the gold standard leads to one set of results in theory, and to quite another set in practice?
  4. “The duty of banks is to act merely as honest brokers between people who have made savings and people who want to use them.” Explain and discuss.
  5. Discuss the relation between changes in long-term and in short-term rates of interest.
  6. Why would you agree, or disagree, with the view that the price-level is the one passive element in the equation of exchange?
  7. How do you account for the course which the prices of imported goods in this country have followed since our suspension of the gold standard in September last?
  8. How far are existing arrangements as to bank reserves adapted to ensure efficient control by the Central Bank over the other banks in the London and New York money markets respectively? What alterations, if any, would you propose?
  9. “The trade cycle constitutes recurrent proof of the absence of tendencies towards equilibrium in the economic system.” Examine the validity of this assertion.
  10. Consider, with reference (a) to past experience, (b) to present circumstances, the relation of the export of capital to equilibrium in our balance of payments.

 

 

Tuesday, June 7, 1932. 9—12.
INDUSTRY AND TRADE.

  1. Show the importance, in past years, of the emigration traffic to the shipping companies of this country and the railroad systems of the New World.
  2. Give some account of the direction and composition of British overseas investment between 1850 and 1900.
  3. “It is obviously to the general interest that sources of supply should grow up as near as possible to centres of consumption; subject to the condition that, where one source has a natural advantage in climate, mineral resources, or deep-set human aptitudes for a particular industry, it may be advantageously developed even at the cost of somewhat large expenditure of labour and material on marketing its products.” (Marshall.)
    Examine, in the light of this statement,
    Either (a) the aggregation of industries in Birmingham and the Black Country;
    Or (b) the location of the milling industry in North America and England.
  4. Define a public utility. How would you approach the problem of whether a particular public utility should be owned and operated by the public?
  5. What evidence would you require to convince you that the organised Produce Exchange renders an indispensable service to the farmer?
  6. Why have the producers of agricultural staples in the New World and in England fared so badly in the last few years? What is the position in continental Europe?
  7. “The 20th century has shown clearly that competitive industry never ends in complete monopoly.” Critically examine this statement.
  8. Seeing that mass production gives decreasing costs, is it not reasonable to suppose that industrial progress and a falling price level will go hand in hand? Consider, in this connection, the price record of soap, automobiles, and electric power.
  9. “Advertisement is a necessary business cost, but of no value to the community as a whole.” Criticise this.
  10. Examine the various causes which may set a limit to the size of a business unit.

 

 

Tuesday, June 7, 1932.  1. 30—4. 30.
LABOUR.

  1. “Our democratic age will be remarkable to posterity for having dimmed the time-honoured belief in the virtues of the poor” (Bosanquet). Is this fair comment on the extension of the Social Services during the past half century?
  2. Discuss the effect of recent changes in the distribution of the National Income on the volume and direction of saving.
  3. Discuss the probable effect on the earnings of workers in the electrical engineering industry of the introduction of a six-hour day coupled with the shift system.
  4. Examine the view that the future of labour organisation lies in Industrial Unionism. Is it supported by the history of Trade Unions in this country?
  5. An industrial combine employing some 30,000 people wishes to introduce a scheme for periodic consultation and discussion between management and employees. Draft an outline constitution. What subjects, if any, would you exclude from discussion?
  6. Give a critical account of the history of government intervention in labour disputes in this country.
  7. Consider the problems involved in the introduction of a means test for claimants to transitional payments under the Unemployment Insurance legislation.
  8. Discuss the decay of apprenticeship. What alternative systems of recruitment are desirable and practicable?
  9. Consider the probable effects of a permanent ban on immigration by the countries of North and South America.
  10. What value do you attach to the suggestions which have been put forward for arresting the growth of unemployment by the speeding-up of expenditure on public works?

 

_________________________

PRELIMINARY EXAMINATION IN ECONOMICS.

Monday, June 5, 1933. 9—12.
PRINCIPLES OF ECONOMICS.

  1. “Natura non facit saltum.” How far can this be regarded as a satisfactory axiom in Economics?
  2. “It is easier to interpret the classical doctrine that ‘Rent does not enter into the cost of production’ in a sense in which it is not true, and to scoff at it, than in the sense in which it was intended and is true. It seems best therefore to avoid the phrase.” Explain and discuss.
  3. State carefully the conditions which must be satisfied in order that an industry may be in long-period equilibrium.
  4. In what circumstances could the distribution of the resources of a community between different uses be improved by government intervention?
  5. On what theoretical grounds can the varying differences between the wages of skilled and unskilled workers in different countries be explained?
  6. “The doctrine that the earnings of a worker tend to be equal to the net product of his work has by itself no real meaning: since in order to estimate net product we have to take for granted all the expenses of production of the commodity on which he works other than his own wages.” Explain the difficulty involved here.
  7. What is the meaning of “normal profits”? How far is it true that profits in all industries tend to reach a uniform “normal” level?
  8. If you had to calculate the most profitable method and rate of exploitation of a mine with a limited supply of ores of varying grades, what factors would you take into account?
  9. Discuss the meaning which is to be attached to the word “utility” in the Theory of Value.
  10. “Increasing and diminishing returns are attributes rather of production as a whole than of production in one individual industry.” Discuss.
  11. In what circumstances if any would you regard it as desirable that a seller should discriminate in the prices charged to different consumers?
  12. “The chief factors determining the rate of interest are the growth of population and the rate of invention.” Discuss.

 

Monday, June 5, 1933. 1½—4½ .
INDUSTRY, LABOUR AND MONEY.

Paper I.

  1. How far is it true to say that the world gold standard before the War was (i) automatic, (ii) managed, (iii) dependent upon the economic and financial policy of England?
  2. “The Quantity Theory of Money throws no light upon the forces which most directly and actively influence the price level.” Discuss this statement.
  3. “The limit of wealth is never deficiency of consumers but deficiency of productive power” (John Stuart Mill). Discuss.
  4. To what extent do you consider that the progressive supersession of private businesses by joint stock companies has been a gain from the point of view of the community as a whole?
  5. Compare, with reference to any industries with which you are familiar, the relative importance of proximity to the market and proximity to the source of raw materials as factors in determining industrial localisation.
  6. Give an account of the difficulties which confront the Lancashire cotton industry in the world markets to-day. What remedies have been attempted or proposed in recent years for improving the position of the industry?
  7. How far does the nature of the product determine the possibilities of success in the co-operative marketing of agricultural produce?
  8. “In disputes between employers and workpeople, the stoppage of work is analogous to a war, or still more to a trade embargo, in disputes between nations.” Discuss.
  9. Give a rough estimate of the proportions of workpeople whose wage-rates are laid down in collective agreements, or fixed by the various statutory wages boards. How far and in what way do these wage-rates affect the levels of wages in unorganised and unregulated trades and services?
  10. Discuss compulsory arbitration in labour disputes from the standpoint of (a) principle, (b) practice, with illustrations from experience at home or abroad.

 

Tuesday, June 6, 1933. 1½—4½.
INDUSTRY, LABOUR AND MONEY.

Paper II.

  1. What would be the probable effects on the economic system of a country which was previously in equilibrium, if the people were to decide to save an additional £100 million per annum and to put the whole of the money thus saved on deposit account with the banks?
  2. Assuming that an important cause of the present depression in England lies in the fact that prices are below the level of costs, consider the relative merits of attempting (a) to raise prices to meet costs, (b) to lower costs to meet prices.

December 1931

December 1932

Treasury Bills outstanding…

£m. 605

£m. 928

Deposits of 12 Joint Stock Banks

£m. 1,806

£m. 2,049

Advances of 12 Joint Stock Banks

£m. 918

£m. 785

Investments of 12 Joint Stock Banks

£m. 298

£m. 488

Bankers’ Deposits at Bank of England

£m. 82

£m. 134

Wholesale prices (1927=100)

65.8

61.1

How do you account for the changes shown in the above table?

  1. Do you consider that discrimination against road transport in favour of the railways would be advantageous to the community as a whole in England to-day?
  2. “‘The optimum firm’ is a meaningless phrase unless we know the price of machinery, the rate of interest and the rate of invention in a given industry.” Discuss this statement.
  3. “The typical manufacturer of the more elaborate goods tends to become little more than an assembler of components bought from specialist makers.” How far is this true? What effects, if any, is it likely to have on labour and industrial organisation?
  4. Why have so many countries been compelled to take special steps for the re-organisation of their agriculture?
  5. In a firm belonging to an industry in which employers and workpeople are well organised, a dispute arises between some of the workpeople and the management as to how many workers, of what grade, and at what wage, shall operate a new type of machine. Describe the kind of agreements that are likely to exist for settling the issue without a stoppage of work, and sketch the procedure.
  6. “Under well organised production the distinction between payment by time and payment by results disappears. The manager who introduces or retains piece-work or bonus systems thereby stands confessed a second-rate organiser.” Discuss.
  7. Discuss the changes in the functions and status of the foreman consequent upon “scientific management” and “labour control,” and the problems such changes create.
  8. Recount briefly the history of any joint industrial council of employers and employed. Discuss the causes of its successes and failures and suggest any reforms you think desirable.

 

Tuesday, June 6, 1933. 9—12.
MODERN ECONOMIC HISTORY.

  1. “Mercantilism was the economic expression of the militant nationalism which sprang out of the social and political changes of the sixteenth century.” Discuss this. Was mercantilism peculiar to England?
  2. Estimate the importance of Spanish silver to the development of capitalism in Great Britain and Europe.
  3. Why was England the mother country of the Industrial Revolution?
  4. Did John Law seriously damage the economic and financial position of France?
  5. Explain the steps by which England originally reached a gold standard.
  6. What was the teaching of Malthus on the subject of population? Was his problem our problem?
  7. Estimate the economic consequences of the Napoleonic regime to Western Europe.
  8. Examine the points at issue between the Currency and Banking Schools in the 1840’s.
  9. Show the relation between the corn law controversy (1813 to 1846) and the formation of economic theory.
  10. How did German thought and policy between 1800 and the death of Bismarck react to English fiscal doctrine?
  11. Give some account of the rise of Socialism on the continent of Europe.

 

Source:  Cambridge University. Economics Tripos Papers 1931-1933, with the Papers Set in the Intercollegiate and Preliminary Examinations 1931-1933. Cambridge, UK: University Press, 1933, pp. 71-83.

Source: Trinity College, Cambridge University. Library of Congress Prints and Photographs Division Washington, D.C.

Categories
Cambridge Exam Questions

Cambridge. Economics Tripos, 1933

 

Examination questions for the Cambridge Economics Tripos from 19211931 and 1932 have been posted earlier. Today we add the 1933 Economics Tripos for good measure.

Economics Tripos Part I (1933)

General Principles I
Social Problems
General Principles II
Essay Subjects
English Economic History
Economic Structure

Economics Tripos Part II (1933)

Economic Principles
Industry
Money
Essay Subjects
Labour
Principles of Politics
Public Finance
The Economic Development of the United States
Statistics
International Law

______________________________

 

PART I.

Monday, May 29, 1933.  9—12.
GENERAL PRINCIPLES I.

  1. “Economics may be described as a scientific study of the best means of avoiding waste.” Discuss.
  2. “Value in all cases tends to equal cost.” Consider the most important ways in which this dictum requires modification or amplification, and bring out clearly what is meant by “cost.”
  3. Is it possible for a community to save and invest too much for its own welfare?
  4. “Since it is evident that any industry is liable in the course of expansion to flood the market with its products and find its profits falling to zero or below zero, it seems pointless to pick out particular industries, such as agriculture, as being special cases requiring the use of the classificatory term ‘Diminishing Returns.'” Comment.
  5. What effects would you expect a great increase in the income of all classes to have upon (a) the rate of interest, (b) the size of the population?
  6. How far is it true to say that capital is free to migrate from declining to expanding industries?
  7. In what way does the true rent of land either resemble or differ from (a) the rent of buildings, (b) the high incomes of successful professional men?
  8. “The rate of interest tends to equality with the marginal net yield on the capital in use.” How does this come about? Is it an adequate explanation of the forces governing the rate of interest?
  9. Why do the prices of some commodities fluctuate much more than others?
  10. What are the chief benefits that might accrue to the community as a whole if a highly competitive industry were compulsorily converted into a monopolistic corporation?

 

Monday, May 29, 1933. 1½—4½
SOCIAL PROBLEMS.

  1. What evidence, if any, is there that poverty has diminished during the present century?
  2. Discuss the difficulties involved in abolishing or modifying the “means test.”
  3. Can relief of unemployment be dealt with on an insurance basis?
  4. Would a uniform national reduction of wages increase employment?
  5. Is England over-populated?
  6. Discuss the proposal to build a million houses in five years and pay for them with new bank credits and notes.
  7. By what chief methods are industrial disputes settled in Great Britain?
  8. Could a general strike improve the position of the wage- earning class?
  9. How do you account for the growing use of piece-rates in Russia since they seem inconsistent with the communist principle, “From each according to his capacity, to each according to his needs”?
  10. Would you support the proposed international convention for a 40-hour week?

 

Tuesday, May 30, 1933. 9—12.
GENERAL PRINCIPLES II.

A.

  1. Discuss the more important ways in which the world with which Economics is concerned differs from that of real life.
  2. Explain what you mean by “the supply of labour” and discuss how far it is affected by changes in the rates of remuneration offered.
  3. What do you understand by the phrase “the representative producer”? For what purposes is the idea useful?
  4. Two commodities are produced under conditions of joint supply. A given tax per unit is laid upon the output of one. Consider the effect upon the price of the other.
  5. What conditions are most favourable to the success of a combination of workpeople in order to raise the price of their labour?
  6. In what ways is the burden of risk in production and trade distributed under present-day conditions?

B.

  1. Give an account of the factors influencing the value of money.
  2. Discuss the difficulties involved in the construction and use of index numbers of the cost of living.
  3. How is the rate of exchange determined between two inconvertible paper currencies?
  4. Describe the working of the London Discount Market and indicate where its importance lies for the student of monetary conditions.
  5. Expand the statement that “the ability of any one bank to extend credit depends partly at least on the policy of the others.”
  6. How far is “the stabilisation of the level of prices” likely to promote the stability of general business conditions?

 

Tuesday, May 30, 1933. 1½—4½.
ESSAY SUBJECTS.

Write an essay on one of the following subjects:

  1. The final goal of society.
  2. “A little college is a dangerous thing.”
  3. Statesmanship and economics.
  4. Hoarding.
  5. The future of the English village.

 

Wednesday, May 31, 1933. 9—12.
ENGLISH ECONOMIC HISTORY.

  1. “Until the end of the seventeenth century England’s economic position was only of secondary importance.”
    Account for this and for the change which took place in the ensuing century.
  2. What was the “yeoman class”? When and why did it decline in importance?
  3. What was the influence of the wars of the Revolutionary and Napoleonic period on the economic development of England?
  4. Outline the events and the discussions which preceded the Bank Charter Act of 1844 and consider the settlement then achieved.
  5. Give a brief account of the “humanitarian movement” and indicate its rôle in the economic and social life of this country.
  6. Give some account of the principal factors which influenced the history of British shipping in the second half of the nineteenth century.
  7. What were the main features of the financial policy of Gladstone?
  8. Indicate and account for the principal changes in the general level of gold prices between 1821 and 1873.
  9. What is meant by “economic imperialism”? What forms did it take in this country in the generation preceding the outbreak of the World War?
  10. Contrast the structure and general situation of British agriculture in 1850 and in 1914.

 

Wednesday, May 31, 1933. 1½—4½.
ECONOMIC STRUCTURE.

  1. “A shift in the geographical distribution of industrial activities is usually associated with technical changes.” Discuss this, giving examples from recent changes in Great Britain.
  2. How do you explain the rapid growth in the proportion of the population employed in the distributive trades?
  3. “In expanding industries productive capacity can be created far more rapidly than of old.” Consider this statement, giving illustrations.
  4. If to-morrow you were appointed dictator of the Lancashire cotton industry for ten years, what would you do? Give your reasons.
  5. Give some account of recent developments in retail trading in this country.
  6. Under what conditions will an increase in the total output of an industry be secured most cheaply (a) by the expansion of existing plants, (b) by the opening of branches, (c) by the establishment of new firms?
  7. In what ways do you consider that public control over the provision of capital for industrial purposes can be improved?
  8. In what circumstances will organised restriction of output in any one industry be in the public interest?
  9. “It may be a true instinct which foresees the decadence of national prosperity in any special change of industry.” Comment.
  10. “Business is becoming a profession which requires a professional training and outlook.” Discuss the truth of this, and its possible effect on the efficiency of management.

 

______________________________

 

PART II.

Monday, May 29, 1933. 9—12.
ECONOMIC PRINCIPLES.

  1. State and criticise the case for the view that the conception of Utility is superfluous in economic theory.
  2. With what qualifications is it legitimate to suppose that the quantity of a commodity demanded at a given place and time is a function of the price of that commodity only?
  3. “The specious symmetry of the expressions ‘law of diminishing returns’ and ‘law of increasing returns’ conceals differences of fundamental importance.” Explain.
  4. What part, if any, do the principles governing the increase of population play in the modern theory of value? If you hold that they play no part, show precisely how population theory has been extruded.
  5. “Collective bargaining can only secure wage advances at the price of unemployment.” Discuss.
  6. Is there any respect in which the laws which govern the rent of land differ from those which govern the value of other factors of production in equilibrium?
  7. On what principles should a socialist state determine the optimum rate of saving for the community?
  8. Construct, if possible, a general definition of normal profit that will cover the cases both of perfect and of imperfect competition. Or, if that is impossible, give specific definitions appropriate to each case. Justify your procedure.
  9. On what grounds would you decide whether a practice of price discrimination was to be condemned?
  10. If a particular factor of production is not fully employed in a community, its employment can normally be increased by a reduction in the price at which it is offered. What relevance has this for the problem of raising output in a community in which none of the factors is fully employed?
  11. What facts would you need to know in order to measure the advantage which a country derived from its foreign trade?
  12. “Monetary theory is destined soon to swallow up the general theory of value.” Discuss.

 

Monday, May 29, 1933. 1½—4½.
INDUSTRY.

  1. How do you account for the increase since the War in the proportion of the occupied population engaged in the distributive trades? Is this increase to be deplored from the economic standpoint?
  2. Do you think it is correct to regard the last quarter of the nineteenth century as a period of stagnation for British industry, and the early years of the present century as a period of recovery? What evidence is available in support of or in opposition to this view?
  3. How would you explain the spread of international industrial combinations since the War? Do the post-War international combinations differ in any important respect from the pre-War international combinations? Have they contributed to economic stability?
  4. It is said that the State regulation of railway charges and services has been developed on the assumption that the railway companies are monopolists. Discuss this contention, and consider how State regulation affects the present distribution of traffic between railways and roads.
  5. What are “secret reserves”? Does the provision of “secret reserves” afford any advantages to business concerns? Can they be justified from the standpoint of the investor and the public?
  6. What is the economic significance of the recent extension of instalment selling? Has this development had an adverse or a favourable effect on the stability of economic life?
  7. Discuss the broad changes in industrial localization within Great Britain during the last hundred years, and their bearing on the theory of industrial localization.
  8. Do you think that the State is ever justified in enforcing rationalization on an industry? What evidence should the State regard as relevant in considering action of this kind?
  9. It is said that certain costs which are variable or prime costs from the standpoint of the employer are overhead costs from the standpoint of the community. Discuss this contention, and consider its bearing on the question of State interference in industrial affairs.
  10. Account for the progressive decline in the British cotton industry during the last decade.
  11. Discuss the causes of the extension of valorization schemes in world agriculture during the post-War period. Estimate the main economic effects of these schemes.

 

Tuesday, May 30, 1933. 9—12.
MONEY.

  1. What light, if any, does the experience of the last three years throw on the comparative merits of the British, German and American systems of deposit banking?
  2. Examine the view that an index number of prices must be differently weighted according as to whether the forces determining the value of money are analysed with the aid of the conception of velocity of circulation or with the aid of the conception of a demand for “real balances.”
  3. How do you account for the popular notion that money, to serve its purpose well, must be “backed by” some particular kind or kinds of asset? Give instances of the expedients which have been adopted to satisfy this notion in times of crisis.
  4. Examine, with special reference to the course of events in the United States in 1922-29, the theory that economic crises are brought about by a deficiency of saving.
  5. “The controversy which culminated in the Bank Act of 1844 was simply an episode in the agelong warfare between inflationist and deflationist.” Discuss.
  6. Examine the nature, and estimate the strength, of the forces which, in the absence of pressure from a central bank, will tend to bring about adjustment in a country’s balance of payments.
  7. Some people hold that, if productivity per head is increasing, the stabilisation by a world monetary authority of an index number of wholesale prices of raw materials would have seriously inflationary consequences. Do you agree with this view? Can you propose any better objective of international banking policy?
  8. The present slump has been compared with that of the early ‘nineties. Explain the main points of resemblance and difference.
  9. If a Government in a slump spends £100 million, raised by loan, on public works, would you expect the “indirect employment” created to last only as long as the “direct employment,” or to outlast it?
  10. You are Finance Minister of an “off gold” country which has decided to re-adhere to an international gold standard. What considerations would you take into account in deciding upon the new parity of exchange?
  11. “Foreign investment on a generous scale by the creditor countries was the root cause of the world crisis: it is also the only possible remedy.” Discuss.
  12. Discuss the influence of central banking operations on the relation between long and short interest rates, with special reference to the maxim that a central bank should endeavour to secure the equality of natural to market rates of interest.

 

Tuesday, May 30, 1933. 1½—4½.
SUBJECTS FOR AN ESSAY.

  1. Japanese Expansion in Asia.
  2. Rivers.
  3. The Economic Background of the Present Situation in Germany.
  4. The Place of Technological and Psychological Factors in Economic Analysis.
  5. Graft.
  6. The Creative Artist in a Socialist State.
  7. The Colour Problem in Africa.
  8. Class War.

 

Wednesday, May 31, 1933. 9—12.
LABOUR.

  1. Compare broadly the standard of comfort attained by the mass of the people in Great Britain with the standards attained in Germany and the United States of America; and briefly account for the differences.
  2. “Unless drastic action is taken to effect a change in the ownership of property, no substantial advance can be made in the direction of equalizing incomes.” Discuss the grounds for this opinion.
  3. Did the legislation passed by Parliament between 1832 and 1850 on balance promote or retard the material progress of the working class? Give reasons for your opinion.
  4. Explain how the volume of employment in Great Britain would probably be affected if the hours of work for all manual workers in industry and commerce were reduced to 40 per week, without any reduction in weekly wage-rates, (a) in this country only, (b) in all countries.
  5. “Experience shows that, while sickness is an insurable risk, unemployment is not, and ought therefore never to have been brought within the scope of social insurance.” Discuss this view.
  6. In a recent enquiry into the Sheffield cutlery trade, relating to the period July—December 1931, information was obtained from 67 manufacturing firms (approximately one-third of those known to be engaged in the trade) selected at random, from the outworkers employed by them and from a number of other outworkers. It showed the following results for men and women of 21 years and over:
Earnings per Hour
(Pence)
Earnings per Week
Upper Quartile Median Lower Quartile Upper Quartile Median Lower Quartile
Time Workers:
Men 15.7 13.7 11.5 61s. 1d. 54s. 10d. 45s. 0d.
Women 6.7 5.9 5.5 26s. 0d. 23s. 0d. 21s. 0d.
Piece Workers:
Men 18.2 14.1 11.1
Women 8.4 6.9 5.4

Of the 2,926 workpeople covered by the enquiry, 61.1% were males and 38.9% females. According to figures supplied by representatives of the trade unions, it would appear that in 1932 less than 20% of the workers in the trade were organized in trade unions. If you had been Minister of Labour, would you have regarded this information as affording sufficient ground for applying the Trade Board Acts to the trade? And how would you have defended your decision?

  1. “To show that wages in a depressed industry with an international market are higher than those paid for similar work in the same industry abroad does not afford sufficient reason for thinking that a reduction of wages is an appropriate remedy for its depression: in order to establish that conclusion it must be shown also that they are higher than those paid for the same skill and efficiency in other industries at home.” Examine this contention.
  2. It has been proposed that a national authority, constituted similarly to the railways’ National Wages Board, should be established to regulate wages and conditions of work in coal mines throughout the country. Discuss the merits of this proposal.
  3. “The differences of structure and policy between the trade union movements of England and the United States are to be traced to historical accidents rather than to any differences in the present character or organisation of the industries of those countries.” How far is this true?
  4. What statistical or other scientific evidence is there as to the effect of (a) noise, (b) rest-pauses, (c) temperature upon output?
  5. Explain, with reference to the history and the underlying economic conditions of the two industries, why the “sliding-scale” principle of wage-settlement worked successfully in the iron and steel trade, but failed to preserve industrial peace in the coal trade.

 

Wednesday, May 31, 1933. 1½—4½.
PRINCIPLES OF POLITICS.

  1. “The Nation, by its power and its duration, is an organism, with an existence, ends and means of action superior to that of the individuals, separate or grouped, who compose it.” (Italian Charter of Labour, 1927. ) Discuss this view.
  2. “The normal course of improvement starts from origination by pure social invention, and passes subsequently, if at all, into adoption by the State” (Bosanquet). Discuss the view of the relation between Society and the State which is implied in this statement.
  3. What are the limits, if any, to the application by the State of the principle of Equality among its members?
  4. “Liberty is not one, but many, and its various forms may contradict one another.” Discuss this statement.
  5. Discuss the idea that law is the enforcement by the community upon its members of a moral minimum of conduct.
  6. How far can an international sovereign authority co-exist with national State-sovereignty?
  7. Would you agree with the view that a system of representative institutions for the economic world ought to be placed by the side of representative political institutions?
  8. On what principles may the State seek to regulate or control the freedom of thought of its members?
  9. On what grounds would you seek to justify the necessity of party and party-organisation for the proper working of Government?
  10. The French Declaration of Rights of 1789 includes the right of resistance among les droits de I’homme et du citoyen. In what sense, if any, can a citizen have a “right” to resist the State?
  11. Discuss, with reference to your general conception of rights, the idea of a “right to work.”
  12. Would you agree that the “separation of the powers” (or functions) of government is a necessary principle of politics?

 

Thursday, June 1, 1933.  9—12.
PUBLIC FINANCE.

  1. Consider the attempts that have been made to frame general rules for Expenditure analogous to those established for Taxation. Account for the relatively backward condition of the former division of Public Finance.
  2. Examine the views that have been put forward concerning the principle according to which motor-vehicle owners should be taxed, and consider the principle or principles to which the present taxes on motor-vehicle owners tend to conform.
  3. Discuss the extent to which the main differences between the various theories of taxation may be attributed to divergent views of the functions and the nature of the State.
  4. What is there to be said for and against having a quinquennial instead of an annual budget?
  5. On what considerations is the maxim that a government should not issue loans below par based? Does the maxim admit of exceptions?
  6. Discuss the view that the Government should spend the revenue derived from death duties on capital purposes.
  7. As a means to increasing prosperity, it has been proposed (a) to reduce taxation without reducing government expenditure, or, alternatively, (b) to increase government expenditure without increasing taxation. Compare the probable effects of these two policies.
  8. “The law of diminishing utility is sound enough when applied to a particular commodity, but it cannot be extended to wealth in general: consequently all the elaborate arguments and formulae for tax-graduation are without logical foundation.” Comment.
  9. Explain exactly what is meant by Most Favoured Nation treatment, and examine the view that its prevalence constitutes an obstacle to the recovery of world trade.
  10. Discuss, with reference to the practice of other countries, possible alternatives to the methods of local taxation at present in force in England.
  11. Is there any foundation for the view that inter-governmental war-debts are more harmful to trade than (a) other debts owed by governments to foreigners, (b) internal war-debts?
  12. Compare the effects, (a) on home producers, (b) on consumers, of an import duty and an import quota.

 

Thursday, June 1, 1933. 1½—4½.
THE ECONOMIC DEVELOPMENT OF THE UNITED STATES.

  1. “America is really a federation of sections rather than of States.” Consider this statement from the economic point of view as applied to American conditions in 1789, 1850 and to-day.
  2. Examine, in the light of the first half century of American history after 1789, the economic powers which the constitution gave to the National Government.
  3. Discuss the policy of the United States Government in dealing with the Public Lands down to the Homestead Act of 1862.
  4. Criticize the methods used by the North to finance the Civil War.
  5. “Southern experience before and since the Civil War condemns slavery as an economic institution.” Discuss this statement.
  6. Illustrate the influence of the westward movement within the continent on the economic outlook of the American.
  7. What problems were created by the consolidation of the railways and what measures were taken as a consequence to protect the public interest?
  8. Account for the leading position acquired by the United States in the iron and steel industry.
  9. Explain the change that has taken place in the twentieth century in the American attitude towards immigration.
  10. To what extent have economic motives influenced the oversea expansion of the United States?
  11. “America has no national economic history, in the sense in which France or England has, until the generation after 1880.” Discuss this statement.

 

Thursday, June 1, 1933. 1½—4½.
STATISTICS.

Part A.

  1. Estimate the average, mode and median for the group of rents in Table A. Criticise the use of the formula skewness = (average—mode) \div standard deviation in this case, and suggest or calculate an alternative measurement.

Table A. Three-roomed tenements.

Rent Number
Under 2s. 6d. 1
2s. 6d. to 5s. 3
5s. to 7s. 6d. 14
7s. 6d. to 10s. 29
10s. to 12s. 6d. 27
12s. 6d. to 15s. 13
15s. to 17s. 6d. 11
17s. 6d. to 20s. 2
100

 

  1. Fit a straight line by least squares or any other method to the series of index-numbers in Table B.

Table B. Index of Production.

Years
Quarters 1927 1928 1929 1930 1931
1st 110 105 108 109 84
2nd 107 103 110 100 80
3rd 105 95 107 90 80
4th 107 104 114 92 90
Average 100

Make a graph of the figures, marking in (1) the straight line found and (2) moving averages based on eight successive quarters.
Comment on the results.

  1. If \bar{x},\,\bar{y} are the averages of n observations of (x, y) pairs, and in the array \bar{x}+{{x}_{s}} at there are ns cases with average \bar{y}+{{y}_{s}}, show that r=\frac{\sum{{{n}_{s}}{{x}_{s}}{{y}_{s}}}}{n{{\sigma }_{x}}{{\sigma }_{y}}}.

Work out r and the correlation ratio for the figures in Table C.

Table C.

Number of earners Number of families Average number
of dependents
x ns \bar{y}+{{y}_{s}}
0 8 1.5 \bar{x}=1.6
1 54 2.1 \bar{y}=2.0
2 19 1.8 {{\sigma }_{x}}=1.13
3 11 2.0 {{\sigma }_{y}}=1.42
4 5 2.2
5 3 2.5
100

Make a graph of the averages showing the regression line. Comment on the utility of the procedure in this case.

  1. From a large population 100 workmen are selected at random and are classified thus:
Occupation
Birthplace Skilled Unskilled
Town of residence 30 38
Elsewhere 10 22

Do you find any relationship between skill and birthplace? Table D may be used if needed.

Table D. Values of P.

{{\chi }^{2}}
Number of Compartments 0 0.5 1.0 1.5 2.0
2 1 0.48 0.32 0.22 0.16
3 1 0.78 0.61 0.47 0.37
4 1 0.92 0.80 0.68 0.57

 

Part B.

  1. What data are used for forecasting the future trend of population and what assumptions are made? Consider also the more limited question of determining whether a population is self-productive or is tending towards diminution.
  2. Among the statistics used for studying the change of the industrial position are (a) an index of physical production, (b) the number of insured persons in employment, (c) an index of wage-rates, (d) occasional accounts of average earnings. What relationships do you expect to find between these statistics in a period of increasing depression or in a period of recovery?
  3. Explain in some detail the construction of index-numbers of average prices and of volume of trade from statistics of exports and of imports. Describe some of the purposes for which these figures are used and consider whether they are sufficiently accurate for these purposes.
  4. Examine carefully the question whether index-numbers of wages and of cost of living can be combined so as to show the relative movements of real wages in different countries.
  5. Describe one of the mathematical methods of constructing “demand curves” from existing data, and consider in what sense they can be called demand curves.

 

Friday, June 2, 1933. 9—12.
INTERNATIONAL LAW.

  1. Comment upon the international status (if any) of the following:

(a) The Dominion of Canada;
(b) The Saar Basin;
(c) The State of California;
(d) Switzerland.

  1. What are the main legal consequences that flow from the independence of States?
  2. Show in what respects a mandated territory differs from (a) a colony, (b) a protectorate. What (if any) is the sanction for the enforcement of the terms of a mandate?
  3. Describe the constitution and purpose of the International Labour Organisation and the method of the composition of the delegations sent by members to its Conferences. Mention any two of the Labour Conventions adopted by it and now in force.
  4. How far has the process of codification been applied to international law? What are the difficulties in the way of further codification?
  5. Describe the three principal methods for the settlement of disputes prescribed by the Covenant of the League.
  6. Discuss the lawfulness and the effectiveness of an economic boycott,

(a) when applied by the nationals of one State against the commerce of another;
(b) when applied by the international community against a wrongdoing State.

  1. Summarise the effect of the outbreak of war upon

(a) treaties to which all the belligerents and no neutrals are parties, and
(b) contracts between the nationals of opposing belligerents.

  1. How much of the Declaration of Paris, 1856, has in your opinion survived the developments of maritime warfare in the Great War?
  2. In what respects do you consider that the Covenant of the League, the Kellogg-Briand Pact, and the Convention on Financial Assistance (1930) have modified the law of neutrality?

 

Source:  Cambridge University. Economics Tripos Papers 1931-1933. Cambridge, UK: University Press, 1933, pp. 53-70.

Image Source: King’s College, Cambridge, England. Library of Congress Prints and Photographs Division Washington, D.C. 20540.

Categories
Cambridge Exam Questions

Cambridge. Economics Tripos Examinations, 1921

 

The following eighteen examinations constituted the Economics Tripos at Cambridge University in 1921. Fifteen exams covered economic theory, policy, history and statistics with three exams covering politics and international law.

Previously posted Cambridge economics exams have been posted at the following links:

Guide to Moral Sciences Tripos 1891
Economics Tripos 1931
Economics Tripos 1932

___________________

ECONOMICS TRIPOS
PART I

Monday, May 30, 1921. 9–12.
GENERAL ECONOMICS. I.

  1. Comment on the following passage: “Les vérités d’économie politique pure fouruiront la solution des problèmes les plus importants, les plus débattus et les moins éclairés d’économie politique appliquée et d’économie sociale.”
  2. “Prices may be determined over a short period by demand and supply; but in the long run they are governed by the superior force, Cost of Production.” Comment on this statement.
  3. What do you understand by the term, Joint Products? Would you apply the term to the production of chocolate and cocoa by the same factory? What principles govern the prices of Joint Products?
  4. “The ordinary progress of a society, which increases in wealth, is at all times tending to augment the incomes of landlords.” Discuss this statement. What effect would you expect a great development of internal transport facilities to exert upon urban rents?
  5. Consider the relative advantages of wages settlements for any industry (a) upon a uniform national basis, (b) upon varying district bases.
  6. Compare the attempts of trade unions to raise wages with those of combinations to raise prices as regards (a) the methods which they employ, (b) the economic conditions of their success.
  7. Distinguish the various types of business which it is customary to describe as Cooperative, indicating in each case the strong and weak points and potentialities of future development.
  8. Comment on the following passage: “General low wages never caused any country to undersell its rivals; nor did general high wages ever hinder it from doing so.”
  9. Is there any inconsistency between the views that the foreign exchanges are determined (a) by the demand for and supply of bills of exchange, (b) by the relative purchasing power of the currencies of the countries concerned? What do you consider the correct view of the matter?
  10. “It is impossible for any country to finance a great war without an inflation of the currency.” Discuss this statement.

 

Monday, May 30, 1921. 1½–4½
RECENT ECONOMIC AND GENERAL HISTORY OF THE UNITED KINGDOM AND THE BRITISH EMPIRE.

  1. Show how Adam Smith’s Wealth of Nations influenced the economic policy of England. What influence had it outside England?
  2. Outline the different measures which have been taken since 1783 for reducing the burden of the National Debt.
  3. What were the main objects which Mr. Gladstone had in view, when selecting the sources from which his revenue was to be derived?
  4. What light is thrown by railway and banking history on the attitude of Parliament towards monopoly?
  5. How far was the distress of the hand-loom weavers and frame-work knitters in the middle of the nineteenth century caused by the introduction of steam power?
  6. What happened to the surplus agricultural population which before the Poor Law of 1834 was maintained, wholly or in part, at the expense of the parish?
  7. (a) “Down to 1870 England’s colonial policy was a policy of drift.”
    (b) “The theorists of 1830 were animated by a mastering passion for liberty, and it is because they succeeded in making it the basic principle of England’s colonial policy that the British Empire is to-day a fact of such cardinal importance in the world.”
    Which seems to you the more correct view, and why?
  8. “The discovery of gold precipitated Australia into manhood.”
    Discuss this; and point out the social problems confronting a new country which possesses precious metals in abundance.
  9. Describe the chief forms of land tenure existing in India, and account for their diversity.
  10. Examine the methods which have been adopted in different parts of the British Empire for the avoidance and settlement of industrial disputes.

 

Tuesday, May 31, 1921. 9–12.
SUBJECTS FOR AN ESSAY.

  1. “Une extrême justice est souvent une injure.”
  2. “In all essentials the problem of industrial peace resembles that of international peace.”
  3.  “The economic relationship of nations is essentially one of cooperation, rather than of rivalry: the prosperity of each country is to the advantage of the others.”
  4. “The nineteenth century was the outcome of French ideas and English technique.”
  5. The Restoration of the Gold Standard.

“From this unrest, so, early wrecked,
A future staggers crazy,
Ophelia of the Ages, decked
With woeful weed and daisy.”

 

Tuesday, May 31, 1921. 1½–4½.
RECENT ECONOMIC AND GENERAL HISTORY OF EUROPE AND THE UNITED STATES.

  1. Contrast the map of Europe, as it was in 1815, with the map of Europe as it is to-day.
  2. To what causes do you attribute the instability of the political institutions of France between 1789 and 1870?
  3. “The German Empire was built more truly on coal and iron than on blood and iron.”
    What evidence is there to support this statement?
  4. “Le Gouvernement provisoire de la République française s’engage à garantir l’existence de l’ouvrier par le travail;
    Il s’engage à garantir du travail à tous les citoyens.” (Moniteur du 26 fevrier 1848.)
    What was the origin of this decree, and to what did it lead?
  5. “Gewiss ist es der Kaiser gewesen und konnte nur der Kaiser sein, der den Fürsten schliesslich entlassen hat, aber die moralische Autorität des Mannes, der das deutsche Reich geschaffen und 27 Jahre an der Spitze der Regierung gestanden hatte, war so ungeheuer, dass es für den Kaiser, der noch so wenig Regierungserfahrung hatte, eine moralische Unmöglichkeit gewesen wäre, sich von ihm zu trennen, wenn nicht eben der Kanzler durch sein Verhältniss zur Majorität des Reichstags sich in eine unhaltbare Position gebracht hätte.”
    Do you agree with this explanation of Bismarck’s dismissal? Give reasons for your answer.
  6. “In France the average unit of agriculture remains as small as ever it was, and its typical manager is still the working peasant or the very small farmer.”
    Why is it that the small farmer has held his own so successfully in France?
  7. Outline the main stages in the growth of the German Zollverein.
  8. Compare the railway systems of Germany and the United States, with particular regard to
    (a) the purposes for which they were planned;
    (b) the part played by the State in their development and control.
  9. How was it that the United States, which Huskisson regarded as our most serious shipping rival, had so small a mercantile marine in the second half of the nineteenth century?
  10. Why were the attractions of Canada, as a field for immigration, inferior to those of the United States during the greater part of the nineteenth century?
  11. What were the chief issues between North and South which led to the American Civil War?

 

Wednesday, June 1, 1921. 9–12.
GENERAL ECONOMICS. II.

  1. “If we wish to get a clear view of English industrial inefficiency at the present time we have only to contrast the average output per worker in American industrial establishments with the average output in English establishments; or to set side by side the average output of each cotton spindle employed in Japan with the corresponding figure for English spindles. In both comparisons England’s position is unsatisfactory almost beyond belief.” Criticise this argument.
  2. What meanings can be attached to the statement that a country is “over-populated”? In what sense, if any, can the statement be applied to this country at the present day?
  3. “The socialistic activities of modern states, by checking the natural play of those forces that tend to bring about the elimination of the relatively unfit, threaten a progressive deterioration of the qualities of the race.” Discuss this statement.
  4. Indicate the chief difficulties in the way of estimating accurately the change in the real earnings of any group of wage-earners in 1921, as compared with 1914.
  5. Are trade unions well advised (a) in attempting to prohibit the entry of female workers into specific occupations, (b) in insisting on equal pay for male and female workers in occupations where both are normally employed?
  6. “Plasticity of wage rates would be the sovereign remedy for the evil of unemployment.” Discuss this view.
  7. Analyse the various causes, both temporary and relatively permanent, which may bring about a disparity between a country’s recorded exports and imports. Illustrate from actual instances.
  8. Discuss the effects of increasing supplies of gold upon the short period and the long period rate of interest in a gold-standard country.
  9. “It is not an essential part of the principle of a gold standard that gold coins should freely circulate within the community adopting that principle, or even that gold coins should be freely minted.” Discuss this statement.
  10. “If all citizens were intelligently alive to their own interests and could also be depended on to be strictly honourable in all their dealings with the State, no taxes on commodities would be tolerated.” Comment.

 

Wednesday, June 1, 1921. 1½–4½.
EXISTING POLITICAL INSTITUTIONS.

  1. “Distrust of the elected legislature is the outstanding feature of the new German constitution.” Discuss this statement.
  2. Compare the functions and the effective powers of the Senate of the United States with those of any one other existing Senate.
  3. Indicate the limitations under which the Referendum is usually instituted, giving reasons for the precautions adopted.
  4. Contrast, in broad outline, the stages by which responsible government has been developed in the Dominions with the steps taken towards its establishment in India.
  5. Describe the relations of the Executive and the Legislature in modern Switzerland. What are their chief advantages and drawbacks? How far could the same system be made applicable in England?
  6. “British and American constitutionalism especially safe- guards the independence of the judiciary in order to ensure the liberty of the subject; European practice aims rather at rendering the powers of the administrators of the law effective against the individual with a view to the safety of the State.” Indicate the extent to which this may be considered a correct statement.
  7. “In America the President and his Cabinet possess executive and administrative functions only; the British Cabinet’s essential functions are of a widely different character.” Comment.
  8. “A written constitution is not the same as a rigid constitution.” Explain this dictum, illustrating from existing constitutions.
  9. Compare Canada with either Australia or the United States as a type of federal state.
  10. In what various ways are the powers of local authorities limited and controlled in the United Kingdom?
  11. “In a unitary state it is impossible to maintain two co-equal legislative chambers.” Comment on this.

 

Thursday, June 2, 1921. 9–12.
GENERAL ECONOMICS. III.

  1. “Eine Grubenabgabe keine Rente ist, mag sie auch oft so genannt werden. Denn ausgenommen den Fall, dass Gruben, Steinbrüche und so weiter praktisch unerschöpflich sind, muss der Überschuss ihres Ertrages über die direkten Auslagen zum Teil wenigstens als der Preis aus dem Verkaufe aufgespeicherter Güter angesehen werden.” Comment on this statement.
  2. “C’est un fait remarquable dans le cas de l’or, que d’un côté les prix des marchandises ayant augmenté depuis 1914 à cause de la diminution sur le marché et pour des autres raisons, la puissance d’achat de l’or à l’égard de ces marchandises a diminué et que d’un autre côté, l’or fait prime parce que de nombreux pays—spécialement les pays d’Asie—montrent une préférence marquée en sa faveur.” Indicate the phenomena thus summarised and investigate the conclusions arrived at.
  3. “It is often said that a high rate of interest hampers production and makes it dearer. This is false.” Examine this view.
  4. Is it to the advantage of a country like Great Britain that its people should invest abroad a large proportion of their annual savings? Discuss the probable economic consequences of attempts to restrict the export of capital.
  5. Examine the effect upon international trade of the payment of a large indemnity by one country to other countries. Is it the case that Great Britain could only receive an indemnity from Germany by increasing her imports of German goods to a corresponding extent?
  6. “A vast unfunded debt is a standing menace to the stability of the banking and currency position.” Discuss.
  7. What data would you require in order to form an opinion as to whether the inhabitants of any European country were under-taxed or over-taxed in comparison with those of other countries?
  8. “Monopolistic combinations promise both steadier levels of prices and steadier conditions for the wage earner than can be expected under the present system of anarchic competition.”
    Discuss this opinion.
  9. “There can be no rise in the value of labour without a fall of profits.” Ricardo. Critically examine this theory.
  10. Discuss the economic significance and consequences of modern advertising.
  11. Comment on the following: “The art of making yourself rich in the ordinary sense is equally and necessarily the art of keeping your neighbour poor.” Ruskin.

 

PART II

Monday, May 30, 1921. 9–12.
ECONOMIC PRINCIPLES.

  1. Write short notes on (a) elasticity of demand and supply, (b) consumer’s surplus, (c) economic rent, (d) quasi-rent.
  2. What determines the selling price of an article under conditions of (a) free competition of purchasers and producers, (b) monopoly?
  3. Examine the relation between the rate of interest and the supply of capital in various classes of investment.
  4. Define “normal profits,” and explain the part played by this conception in Marshall’s theory of distribution.
  5. Consider the advantages of fixing wages on a sliding scale based (a) on the cost of living, (b) on the selling price of the product of the industry, (c) on the aggregate output of the industry. Give instances of each method.
  6. Consider the arguments for and against a countervailing import duty on goods imported from a country, of which the currency is greatly depreciated in terms of the currency of the importing country.
  7. Would it be beneficial if a State Bank, having the right to issue inconvertible notes, were to make loans for all approved purposes without interest, provided the borrower was able to furnish war loan or similar stock with a 10 per cent, margin as security? Frame your answer so as to persuade a person inclined to hold the opposite view to your own.
  8. What would be your test of over-population? Apply it in a general way to present conditions in India, Germany, England and the United States.
  9. “On peut trouver entre les faits d’ordre économique des relations de dépendance que l’on peut essayer d’exprimer par des formules algébriques, alors même qu’on ne pourrait pas les traduire en chiffres.”
    Give three or four examples of this.
  10. “Kapital ist der Teil des Vermögens, welcher, selbst Produkt menschlicher Arbeit, wieder zur Produktion bestimmt ist. Nach dem Sprachgebrauche liegt sowohl der Begriff des Vorrats für künftige Nutzung, wie der des Erwerbsmittels darin doch erscheint es richtiger, das Kapital im Gegensatz zur Natur als bestimmten Teil des Vermögens hinzustellen und den Nachdruck auf die Art der Verwendung zu legen, welche die Güter finden sollen, weil sich so die schärfste Abgrenzung des Begriffs durchführen lässt. Es sind nicht darunter zu begreifen die Güter, deren Wert nicht geschätzt werden kann, wie persönliche Fähigkeiten, der Staat usw. und die freien Güter.”
    Restate the salient points of this definition in your words, and point out the respects in which a different view has been or might be taken.

 

Monday, May 30, 1921. 1.30–4.30.
THE ECONOMIC FUNCTIONS OF GOVERNMENT.

  1. “Now that Public Finance is deliberately used as an engine for the redistribution of wealth, the conception of justice in taxation has become meaningless.” Examine this view.
  2. On what principles should the cost of elementary education be distributed between the National Exchequer and the Local Authority concerned?
  3. Consider the case for the State endowment of motherhood.
  4. “Taxes on business transactions are open to all the same objections as taxes on commodities, and should be entirely abolished.” Consider this contention.
  5. Investigate the view that even in ordinary times the State should control the price and ration the supplies of essential articles of food.
  6. How far can the allocation of public funds be made in such a way that the marginal pound will yield equal advantage in each direction of public expenditure?
  7. Discuss with illustrations the obligation which rests on a modern State to take steps to conserve the wasting natural resources contained within its borders.
  8. “A municipality should not undertake to supply any article or service unless it is prepared to become the sole supplier.” Do you consider that this advice is sound?
  9. “In einer Zeit, in der man eifrig nach neuen Steuereinnahmen Umschau zu halten hat, ist es begreiflich, dass insbesondere angesichts der Umwerfungen der Werte in rasch aufblühenden Gemeinwesen nicht nur bei radikalen Bodenreformern, die zunächst eine Verstaatlichung des Bodens, eine Wertzuwachsteuer immerhin als Abschlagszahlung wünschten, sondern über diese Kreise hinaus der Ruf ertönte nach einer Steuer die 1. mühelos erworbene Gewinne für die Gesamtheit möglichst stark mit Beschlag belegen sollte, 2. auf die Benützer des Bodens nicht überwalzbar erschien.” Discuss.
  10. “Entre les trois procédés de couvrir les dépenses de la guerre par l’impôt, par l’emprunt, ou par l’association des deux, M. Helfferich est demeuré bien en arrière de ses collègues d’Angleterre et même de Russie. L’Angleterre a couvert par les recettes de l’impôt 25% des énormes dépenses de guerre.”
    Consider the arguments for the various policies pursued by the belligerents in this respect.

 

Tuesday, May 31, 1921. 9–12.
STRUCTURE AND PROBLEMS OF MODERN INDUSTRY.

  1. “The liner she’s a lady.”
    “The black Bilbao tramp.”
    Explain the problem of ocean freights, as between these two services.
  2. Discuss the advantages of the holding company and the merger respectively as methods of combination, (a) with reference to industrial efficiency, (b) from the financial standpoint.
  3. Consider the problem of “Key industries,” (a) as to definition, (b) as to a right policy of development.
  4. Describe the prevailing systems of land tenure in England. What is there to be said in favour of the landlord?
  5. “Unfair methods of competition are hereby declared un- lawful.” (Federal Trade Commission Act, Sect. 5.)
    By what principles should a Court be guided in enforcing this clause? Illustrate by reference to special practices.
  6. What features of the existing structure of industry lead to proposals for the limitation of profits ? Give instances of such proposals, and discuss their effects.
  7. Suppose a scheme to be established for the benefit of operatives, e.g. in the cotton or in the engineering industry; how would you define the “industry” for this purpose? Show by examples the problems which would arise.
  8. Compare the organisation of the Cotton and the Wool Textile Industries as regards (a) the market for the raw material, and (b) the market for the finished articles.
  9. On what grounds has the present organization of the British coal industry been criticised from the standpoint of efficiency?

 

Tuesday, May 31, 1921. 1.30–4.30.
DISTRIBUTION AND LABOUR.

  1. Examine the probable effect on the real incomes of house-owners if the cost of building increases, other prices generally remaining unchanged, (a) when there is no legal restriction on rent, (b) under such an act as is now in force.
  2. Suppose a gradual transference in income, owing to rising prices and wages, from owners of securities bearing fixed rates of interest to workmen. How is the rate of saving affected, and what are likely to be the ultimate effects on national income if the transference results in a higher standard of living for workmen?
  3. Discuss the incidence of compulsory provision of insurance against (a) unemployment, (b) sickness, if the contributions are paid in part by the state, in part by employers and in part by wage-earners.
  4. Examine the conditions under which the establishment of a minimum wage in an industry does not result in diminution of employment, illustrating your answer from the experience of Australia or of the Trade Boards Acts in the United Kingdom.
  5. Among the effects of the wage-changes in 1915 to 1920 were the diminution of inequalities of wages between industries, between occupations and between localities. How far were such inequalities due to economic causes and how far to custom or to accident?
  6. A workman’s needs generally increase between the ages of 20 and 40, while in many occupations wages are irrespective of age. Can any methods be suggested by which this anomaly can be avoided?
  7. The causes of poverty can be roughly classified as economic or personal. Indicate some methods by which trade organisation can remove the former and state organisation the latter.
  8. Analyse the difference between the methods of settling wage disputes by compulsory arbitration, voluntary arbitration, and such organisations as Whitley Councils, giving illustrations from the practice of this or any other country.
  9. During the generation before the war there is evidence of a steady increase of the number of salaried persons relatively to the number of wage-earners. Consider the effects of this tendency on the statistics of the growth of the national income as they are usually classified.

 

Wednesday, June 1, 1921. 9–12.
MONEY, CREDIT, AND PRICES.

  1. Write a brief essay on the Quantity Theory of Money, describing in outline at least two methods of approach, and stating which you yourself prefer.
  2. State the chief theoretical difficulties in compiling an index number of the cost of living which shall be valid for comparisons between different places over a considerable period of time. Is there any important theoretical distinction between “cost of living” index numbers and index numbers of the value of money generally?
  3. Analyse and compare the different ways in which the term inflation is used by the wise and the foolish.
  4. How do you explain the fall of prices during the past twelve months? Do you expect it to continue?
  5. Compare the position of the U.S. Federal Reserve Board in the banking system of the U.S. with that of the Bank of England in the British system.
  6. Give an account of the probable career from start to finish of a cotton bill drawn to finance the purchase of American cotton by a Liverpool merchant.
  7. Give a careful account of the chief factors which influence the foreign exchanges, and forecast, with your reasons, the probable level of the dollar, franc and mark exchanges six months hence.
  8. Restate the theory of international trade for the case of a set of countries of which the currencies are not on a gold basis but consist of unrelated inconvertible paper moneys.
  9. Make practical proposals for obtaining from Germany as large as possible a sum in payment of Reparation.

 

Wednesday, June 1, 1921. 1.30–4.30.
SUBJECTS FOR AN ESSAY.

  1. The Money Motive.
  2. Business men in politics.
  3. Frontiers.
  4. Types of national leadership in Industry.
  5. Education and Industry.
  6. Bolshevism.

 

Thursday, June 2, 1921. 9–12.
MISCELLANEOUS ECONOMIC QUESTIONS.

  1. “Light is the sovereign antiseptic, and the best of all policemen.” Do you regard this maxim as an adequate guide for the State to follow in its dealings with industrial combinations?
  2. Consider how far the ordinary arguments against artificial interference with prices are relevant to arrangements for controlling the price and rationing the supplies of bank loans.
  3. “The great national Trade Unions, by their insistence on uniformity of conditions, are really hindering the workman in his attempt to better his lot and to secure a greater share in the government of industry.” Comment on this opinion.
  4. Examine the view that there is a danger of experiments in scientific management being carried further than the true interest of the nation demands.
  5. “The policy of giving subsidies to unemployed workmen necessarily involves inflation of currency, and therefore aggravates the evils it is designed to cure.” Discuss this statement.
  6. In what ways, if any, could the State usefully intervene to check the wastes caused by competitive advertisement?
  7. Examine the arguments for compelling railway companies to carry workmen going to and from their work at unremunerative rates.
  8. Consider the proposal that wage agreements negotiated by bodies representing the majority of employers and employed in any trade should be made legally binding on the remainder of the trade.
  9. Can you suggest any practicable scheme for the issue of an international currency by the League of Nations, in order to facilitate the resumption of international trade?

 

Thursday, June 2, 1921. 1.30–4.30.
THE THEORY OF STATISTICS.

  1. “The precision of the average of the measurements of n objects selected at random from a large group is proportional to \sqrt{n}.” Explain carefully the meaning of this statement, discuss the conditions under which it is true, and give examples of its use.
    What modification is needed if n/N is not negligible, where N is the number of objects from which selection is made?
  2. How does the analysis, which gives the ordinate of the normal curve of error as the limit of nCpn+xppn+xqqn-x when n is great, break down if pn is not great, and what is the resulting form?
  3. Distribution according to age of railway guards.

Age

Number of persons per 1000

Under 15 years

0

15—

8
20—

48

25—

252

35—

334
45—

223

55—

125
65—

10

75 and over

0

1000

Estimate by any method of interpolation the relative numbers in the age groups 30 to 35 and 35 to 40 years. State the hypotheses on which the method rests.

  1. For the frequency group in question 3 calculate the average, median and mode, and obtain one measurement of deviation and of asymmetry.
    Give a verbal description of the group, using your results, in language which avoids technicalities as far as possible.
  2. A six-faced die is thrown seven times; the numbers of the pips shown in the first five throws are added (x) and also of those shown in the last five (y). Show that if the experiment is repeated the value of the product-sum coefficient of correlation between x and y tends to be 3/5.
    Discuss the question whether any useful meaning can be given to the coefficient of correlation (obtained by the product-sum formula) in a case where nothing is known of the genesis of x and y and their distribution is not normal.
  3. Describe any one method by which association between non-measurable attributes (such as intellectual ability of pairs of brothers) can be tested, and consider whether a valid numerical measurement of association can be obtained.
  4. Find (by any method) the equation of a regression plane applicable to the following table, regarding each of the 15 entries as of equal importance.
    Compute the corresponding values from the equation and comment on the differences between the observed and computed numbers.
    What further information would be needed to calculate regression coefficients’?

    Age of wife at marriage

    Children born per 100 couples by age of husband
    15 – 25 25 – 35

    35 – 45

    15–20 years

    813 714 629
    20–25 661 595

    540

    25–30

    498 462 434
    30–35 364 341

    322

    35–40

    171 160

    150

  5. In a certain population the number of males over 25 years old is 875712, and the numbers who were 25, 26, 27, 28, 29 years last birthday respectively 29556, 29336, 29078, 28816, 28542. From these data, fill in the following, where the letters have the meanings usual in life tables:

    Age

    lx Lx dx qx

    mx

    24

    26

    27

    28

    29

If qx is the rate of mortality, mthe central death rate, and \mu_{x} the force of mortality, show that q_{x}= \frac{2m_{x}}{2+m_{x}} and approximately \mu_{x}= m_{x-\frac{1}{2}}.

  1. What data are necessary for measuring how far a low birth-rate is compensated by low infant and child mortality? Show in some detail the form of statistical results obtainable.

 

Thursday, June 2, 1921. 1.30–4.30.
ECONOMIC CONDITIONS IN ENGLAND 1823-1828,
AND CONTEMPORARY SOCIAL THOUGHT.

  1. Give an account of Place’s efforts to defeat the reactionary intentions of the Government in 1825 with regard to the Combination Act of 1824.
    How far is he justified in his statement that “Ultimately the Act (of 1825) differed very little from Mr Hume’s Act. It is substantially the same”?
  2. On what grounds did Huskisson advocate
    (a) the retention of the “long haul” clause in the Navigation Act,
    (b) imperial preference?
    Was the U.S.A. justified in regarding the latter as a continuance of the old tariff discriminations?
  3. “The question, then, looking at it practically, is this: In what degree is Prohibition better, as against smuggling, than a well-regulated duty?—by which I mean, a duty sufficient to protect the British manufacturer, without being so high as to afford a premium to the smuggler.” (Huskisson: Speech on the Silk Manufacture, Feb. 1826.)
    Show how this consideration guided Huskisson in the changes which he made in the customs tariff.
  4. What were the leading ideas in the programme of the early London Cooperators, and from what source or sources were they derived?
  5. Why were the problems of the Irish Poor and the English Hand-loom Weavers treated by the Emigration Committees of 1826 and 1827 as a single problem?
  6. Show how the fear of a redundant population dominated the recommendations of the Emigration Committees of 1826 and 1827.
  7. “In Lancashire there appear to be among the Hand-loom Weavers two classes almost wholly distinct from each other; the one who, though they take in work in their own homes or cottages, are congregated in the large manufacturing towns; and the other, scattered in small hamlets or single houses in various directions throughout the manufacturing country.” (Emigration Committee, 1827.)
    Was this distinction of long standing? Was it peculiar to Lancashire?
  8. Illustrate from the Rural Rides Cobbett’s first-hand acquaintance with agricultural conditions.
  9. “The Ricardian Socialists, though fairly unanimous in their criticisms of the existing system, differed greatly as to remedies.”
    Illustrate this from the writings of William Thompson, Thomas Hodgskin and J. F. Bray.
  10. If you were engaged in an investigation into social conditions in England (1823-8), which are the trades about which you would find it most difficult to obtain information, and why?

 

Thursday, June 2, 1921. 1.30–4.30.
INTERNATIONAL LAW.

  1. Describe the system of Colonial Mandates in the Covenant of the League of Nations. Indicate what are the important changes introduced and, in particular, consider the question of the sovereignty over the mandated areas.
  2. How far do you consider that the principle of “Self- Determination” has a basis in International Law?
    Comment on the following statement of Mr Lansing: “It is an evil thing to permit the principle (i.e. of Self-Determination) to continue to have the apparent sanction of the nations when it has been thoroughly discredited.”
  3. Compare the position in International Law of the Suez and Panama Canals.
  4. Examine the following statement of Lord Stowell: “In my opinion there exists a general rule in the maritime jurisprudence of this country by which all trading with the public enemy, unless with the permission of the sovereign, is interdicted. It is not a principle peculiar to the maritime law of this country.”
    On what grounds is such a principle to be justified?
  5. Comment on one of the following passages:
    (a) “II y a donc, voulue par la conscience universelle, une certaine limitation aux moyens d’hostilités, limitation encore bien imparfaite et qu’il est ambitieux de décorer du nom pompeux de lois de la guerre. Disons, plus modestement, qu’il existe des usages de la guerre dont le droit international demeure le guide moral et l’inspirateur.”
    (b) “Gegenüber der rechtmässigen Ausübung des Anhaltungs-, Durchsuchungs- und Wegnahmerechts gibt es für das Kauffahrteischiff kein Recht der Notwehr. Die Notwehr ist begrifflich die Verteidigung gegenüber einem rechtswidrigen Eingriff in ein Rechtsgut. Dies gilt allgemein für die neutralen wie für die feindlichen Kauffahrteischiffe. Die letzteren nehmen keine Ausnahmestellung ein. Auch sie besitzen kein Recht der Notwehr.”
  6. To what treatment are merchant ships belonging to the citizens of a belligerent state which are in the ports of the other belligerent at the outbreak of war subjected by International Law?
    What course was taken by Great Britain, with regard to German and Austrian ships in British ports at the outbreak of the war in 1914?
  7. Discuss the origin and development of the doctrine of “Continuous Voyage.”
  8. Do you consider that any violation of neutrality has been committed in any of the following cases?
    (a) The export of munitions by neutral manufacturers to one of two belligerents.
    (b) The granting permission by a neutral state for the passage through its territory of the sick and wounded of a belligerent’s army.
    (c) The granting of permission by a neutral state to belligerent warships to effect repairs in its ports.
  9. Discuss the propriety of the use of the term “Blockade” in relation to the measures taken by Great Britain under the Order in Council of 11 March 1915.
  10. What is the effect of lapse of time upon the binding force of a treaty?
    Discuss the action of Austria in annexing Bosnia and Herzegovina in 1908.

 

Friday, June 3, 1921. 9–12.
POLITICAL SCIENCE.

  1. “By Liberty I mean the assurance that every man shall be protected in what he believes his duty, against the influence of authority and majority, custom and opinion.” (Acton.)
    Use this opinion to show how you define Liberty.
  2. How has the argument regarding the rights of property been affected by the development of modern industrial conditions?
  3. State and examine Rousseau’s distinction between the General Will and the Will of All.
  4. Discuss Mill’s view of the conditions essential to the success of representative government.
  5. What are the conditions or grounds of objection which give a minority the strongest right to refuse obedience?
  6. On what principle should the relations between a first and a second chamber be regulated in a modern democracy?
  7. Give a critical estimate of the position which has developed between the State and Labour, through the Social Reform movement of the last generation in England.
  8. Explain the alternative implied in Weltmacht oder Niedergang. What reactions on the life of the great State have been attributed to the development of colonial possessions?
  9. What are the main considerations which determine a sound theory of punishment?
  10. “The separation of powers in politics corresponds to the fixity of species in natural science; and both ignore evolution.”
    Comment on this statement.

Source: Cambridge University Economics Tripos Papers, 1921-1926. With the papers set in the Qualifying examinations 1925 & 1926. Cambridge at the university press 1927, pp. 1-21.

Image Source: Kings College, Cambridge England. Library of Congress Prints and Photographs Division, Washington, D.C. 20540.

Categories
Cambridge Chicago Teaching

Chicago. Harry Johnson’s observations and reflections on teaching, 1969

 

The transcribed letter below was written by Professor Harry G. Johnson to (then) graduate student Michael Mussa whose proposal for student evaluations of graduate courses at the economics department of the University of Chicago met with hostile reception. It is always a genuinely nice gesture for a senior professor to take time and effort to recognize a student initiative and this letter is a model of such a response. I presume the copy of this letter I found in Milton Friedman’s papers had been shared by Johnson with his colleagues.

Johnson first reflects on the nature of teaching in a leading graduate institution, concluding on the one hand that some bad teaching will be inevitable but that even the worst teachers could improve their (literal) performances. He then illustrates with his own course (the third of the three quarter sequence in Price Theory) followed by three examples from his own Cambridge training: D. H. Robertson, Maurice Dobbs, and Joan Robinson. 

Pro-tip: the snap-shot of Harry Johnson comes from Robert J. Gordon’s very own personal collection “Photos of Economists”.

Johnson mentioned that Joan Robinson refused to hand out reading lists for her courses. For a rare Joan Robinson reading list: from Williams College 1982.

_________________

From a Photocopy of a letter from Harry Johnson to Michael Mussa

May 28, 1969

Michael Mussa:

I’ve been reading the reports of the faculty-student advisory committee, in which your proposal for student evaluations of courses seem to have fared rather badly. Personally, I am more favorable to the idea than my colleagues seem to have been.

There are of course plenty of problems in defining the function of such evaluations; and at least in a place like Chicago teaching competence can’t be given much weight (a) because a large part of staff work is guiding Ph.D.s and conducting the workshops—i.e. research rather than teaching oriented; (b) because our courses ought to be not only teaching the accepted structure of knowledge, but exposing students to the frontiers of the subject, and this kind of material is often difficult to teach; (c) our competitive position as a leading world graduate school, which among other things determines the quality of the students we get and therefore the “externalities” our students obtain from each other, ultimately depends on the published scientific contributions of our faculty and not on their capacity to teach a particular course well at a particular time.

On the other hand, I do not share the view of some of my colleagues—both here and even more in England—that teaching performance is an absolutely fixed characteristic of the individual that cannot be altered by care and study on his part. Consequently I think that student evaluations can be useful to both the individual teacher and his colleagues, as guides to where some investment in improvement could usefully be undertaken.

One obvious point, with respect to which I benefitted from last year’s evaluation, is the course reading list. As a result of that evaluation, I took off the 302 reading list an article by Champernowne, the approach of which I wanted represented in the course, but which students considered too difficult for what they got out of it; instead, I now present the approach in very simple form. I also took off a number of readings on the poverty problem which were significant when the war on poverty started, but which now appear to have little substantive content. I now link the poverty material more closely to the general theme of the course.

I think that student evaluation of the reading list can be very valuable in indicating what readings are really useful and what are either too hard or too easy for the level of the course. An even more important function, which would be harder to devise, would be for students to suggest from their own current and previous reading of the journals and textbooks better sources for the main parts of the course. The literature is tremendous, and there is no easy way of searching it for the most useful contributions.

As to the teaching itself, it seems to me that there are two separate problems, each of them susceptible of solution by rational investment by the individual teacher. The first is organization of the material; this includes reading list and course organization, organization of the individual lecture and supplementation of the lecture by appropriate hand-outs of crucial data or pieces of analysis. The Chicago course-load is fairly light by most universities’ standards, and the purpose of the lecture I teaching rather than public virtuoso performance. There is little reason why a person who knows or is told that he is disorganized and confusing while on his feet in front of a class should not provide his students with the insurance of a paper version of what he meant to tell them. The second problem is that of personality. A teacher has to understand that in the classroom he is playing a role, which does not have to define or exhaust the full scope of his private personality; and he has to forego the tricks that people use in private conversation to defend themselves and preserve what they think is the respect of others. Specifically, it is not helpful to students to be exposed to an account of all the mental confusion that accompanied the first discovery of a new truth (especially if the truth itself never emerges). Nor does it help for a lecturer to use two methods of disguising uncertainty or insufficient thought in private conversation: (1) vehement assertion of a conclusion without adequate supporting argument; (2) dropping the voice just as the crucial point is reached, so that the audience doesn’t really hear what is being said (Margaret Reid picked me up on this trick when I first came to Chicago).

It seems to me that lecturers have a lot to learn from the acting profession in this respect: even if the lines are lousy, they deliver them with conviction. It also seems to me that student evaluations would help to tell lecturers that, however good the message they thought they were putting across, it was being scrambled in transmission and wasn’t reaching the students.

I would not pretend that everybody can be a good lecturer. All I claim is that most people could do better than they do, by recognizing their weaknesses and trying to correct them. Even so, you could well not get very good lectures.

I remember when I was a student at Cambridge, England. D.H. Robertson was Professor; he had had a lot of acting experience in his youth. He wrote every word of his lectures, and rewrote them every year. He delivered them well, but you had to listen hard and know a lot already to get the points. Yet he would never answer any questions from class; once a year, in the second-last lecture, he would ask the class for written questions; in the last lecture, he would read us his written answers to the written questions. Maurice Dobb, who was my supervisor and whose lectures I attended religiously out of a youthful enthusiasm for left-wing causes, also wrote every word of his lectures on socialist planning, and they were beautifully logical and well-organized constructions; but he read them in a flat monotone that rapidly depressed the audience, with the result that the beachhead of communism he established in Cambridge never got occupied by troops prepared to push on to a major assault on capitalism. Joan Robinson, on the other hand, always claimed she lectured as the spirit moved her, and refused to give out either a reading list or reference (apart from insulting remarks about D.K. Robertson, and favorable references to Keynes and Kalecki). In fact she said the same thing every year; but the students always got excited because to them it came as something new.

In my judgment, Robertson could have learned to answer the questions he could answer, and ask for time to consider the others; Dobb could have learned to modulate his voice to emphasize the difference between major points and supporting arguments; and Robinson could have been persuaded by student demands to produce a reading list. But it is quite likely that none of them would have changed their ways; and I doubt that Cambridge would have been well advised to fire them if they hadn’t.

Yours sincerely,
[signed]
Harry G. Johnson
Department of Economics

HGJ/sf

 

Source: Hoover Institution Archives. Papers of Milton Friedman. Box 194, Folder “194.4 Economics Dept. A-G”.

Image Source:  Harry Johnson. Photo by Robert J. Gordon, Summer 1970 or 1971.

 

Categories
Cambridge Chicago Columbia Economists Harvard Ohio State Vanderbilt

Harvard. Economics Ph.D. alumnus, James W. Ford, 1954.

 

In this latest addition to our series “Get to Know an Economics Ph.D.”  we meet a Harvard Ph.D. from 1954, James William Ford.  His Ph.D. dissertation’s title was “International monetary relations and the British monetary system, 1920-1939”.

Ford’s academic path began as an undergraduate at Oberlin, then he went on to Harvard for his graduate work. Before getting his Ph.D., Ford received one of the very first round of Fulbright Fellowships to attend Cambridge University. He taught at Columbia, Vanderbilt, and Ohio State followed by two years working at the Board of Governors of the Federal Reserve System. His long final career stage was with the Ford Motor Company as a leading financial economist.

_____________

James William Ford (1923-2017)
Obituary

James William Ford, a beloved father, grandfather, and great-grandfather, died at age 94 on November 23 at his home in Ann Arbor, Michigan. Mr. Ford was born February 1, 1923 in Alameda, California, the son of Eunice George Ford and Shelton C. Ford, and older brother of Eunice Ford. He is survived by his second wife, Phyllis Ford; three children, Julian Ford, Amy Milkovich, Carol Arkin; two step-children, Jessica Leix and Peter Leix; 10 grandchildren; 1 step-granddaughter; and 7 great-grandchildren. In the first three decades of his life, Mr. Ford was an outstanding student and a City of Detroit High School Debate champion, served in the Army as a meteorologist during World War II, a graduate of Oberlin College in 1947, a Master of Arts recipient in economics from Harvard University in 1949, one of the first class of Fulbright Scholars in 1951 (at Cambridge University in Great Britain), and Doctor of Philosophy recipient in economics from Harvard University in 1954. Mr. Ford taught economics at Columbia University from 1951 to 1953, at Vanderbilt University from 1953 to 1957, and Ohio State University from 1957 to 1959, before becoming a postdoctoral fellow at the University Chicago with the eminent economist Milton Friedman. Mr. Ford served as Economist to the Board of Governors at the U.S. Federal Reserve from 1959-1961. He then moved to Ford Motor Company where he worked for the rest of his career until retiring in 1988. Mr. Ford was the Assistant Controller for the Ford Motor Company Finance Staff from 1961 to 1975, Executive Vice President for Insurance and Special Finance Operations at Ford Motor Credit Company from 1975-1977, then president from 1977-1980 and Chairman,1980-1987, of Ford Motor Credit Company. At Ford Motor Company he became Vice President from 1980-1987, Executive Vice President from 1987-1988, and President of Ford Finance Services Group from 1987-1988. Under his leadership, Ford Motor Credit Company developed a program and portfolio of financial policies and investments that achieved unprecedented fiscal success for the company. He visited and met with Ford Motor Company dealership executives all over the country, developing a network of successful entrepreneurs and many close friendships that lasted throughout his retirement. After retiring at age 65, Mr. Ford was very active for the next 25 years as a Board member for several nonprofit agencies serving children and families, investment firms, and most especially with the United Methodist Retirement Community and the Towsley Center in Chelsea, Michigan, where a wing is dedicated to his mother and a garden is dedicated to his beloved first wife Anne, and with Starfish Family Services. Mr. Ford was an avid tennis player for most of his life and captained a small sailboat every weekend for many years, and followed in his mother’s tradition by traveling widely around the world. He was a devoted brother to his younger sister, Eunice, and was much loved by many other members of the Ford family and in-laws on the Farley side of his and Anne’s family, and countless close friends including members of a potluck group in Ann Arbor that convened monthly for more than four decades. According to his wishes, a gravesite service will be held at Botsford Cemetery in Ann Arbor in the Spring…

Source:  Published in Ann Arbor News on Dec. 3, 2017.

Image Source: Oberlin College Yearbook, The Hi-O-Hi, p. 32.

Categories
Cambridge Exam Questions

Cambridge. Examination Questions of the Economics Tripos. 1932

 

In the U.S. Library of Congress I came across a collection of the Cambridge University Economics Tripos examinations for 1931-1933. In an earlier post I provided transcriptions of the 1931 exams. This post provides the 21 examinations for 1932. For a later post I’ll transcribe the 1933 exams.

___________________

PART I.

Monday, May 30, 1932. 9—12.
GENERAL PRINCIPLES I.

  1. Bring out clearly the economic principles which determine the relative values of a bushel of wheat and a pair of boots.
  2. Define the following and consider the relations between them: wealth, capital, land.
  3. What do you understand by (a) saving; (b) investment? Explain the process by which savings are converted into capital.
  4. A tenant farmer is cultivating a dairy farm in the West of England and paying a rent of £200 per annum. The landlord dies and the farm is put up for sale and offered for £4000 to the tenant, in the first instance. What various economic factors should influence the farmer in deciding whether to buy or not?
  5. What forces tend to remove and what forces tend to perpetuate inequalities in wage rates (a) in the same industry in different parts of the country; (b) in different industries in the same country?
  6. Bring out the importance for the theory of value of (a) marginal utility; (b) the principle of substitution.
  7. How do you account fort he observed tendency for the prices of foodstuffs and raw materials to fluctuate much more widely than the prices of most manufactured articles?
  8. Under what circumstances is it likely that unrestricted competition will lead to the formation of a monopoly?
  9. On what does the marginal net product of labour depend?
  10. Trace the stages by which a sudden but enduring fall of 50 per cent. in the demand schedule for an article will react upon (a) the supply of that article; (b) its price.
  11. Analyse profits on capital and discuss the tendency of each element to rise or fall.
  12. “To use the phrase ‘negative quasi-rent’ is to misconceive the nature of Marshall’s doctrine of ‘quasi-rent.’” Comment on this statement.

 

Monday, May 30, 19932. 1½ — 4½.
SOCIAL PROBLEMS.

  1. What do you understand by (a) primary poverty, (b) secondary poverty? Are there in your view any immediate means of removing the causes or remedying the effects of primary poverty?
  2. Which is the more suitable body for applying the “means test” to applicants for unemployment benefit, the Ministry of Labour or the Public Assistance authority? Is the test desirable as a permanent feature of the unemployment insurance system?
  3. In which industries is unemployment most severe at the present time? In the event of a general revival of trade in England in the near future, in which industries would you expect employment to increase most rapidly?
  4. Compare with reference to the course of events since the War the relative efficacy of (a) “direct action,” and (b) political pressure for increasing taxation and social services, as methods of improving the welfare of the working classes.
  5. There has recently been a widespread substitution of piece-work for time-work in Russia, leading to large increases of output. Would you regard this as evidence in favour of extending piece-work in other countries?
  6. Consider the arguments for and against the extension of unemployment insurance to cover agricultural workers.
  7. What evidence is there that the population of Great Britain will begin to decline within twenty years? Would a declining population solve (a) the housing problem, (b) unemployment?
  8. Would you favour the use of a Government housing subsidy to provide rent rebates varying according to the size of the family housed?
  9. Under what conditions will the general introduction of the automatic loom in Lancashire aggravate unemployment? Is it desirable to introduce labour-saving machinery during depression?
  10. How would you account for the growing tendency in recent years for the formation of Industrial Unions rather than Craft Unions?
  11. “It is a direct corollary of the ‘marginal productivity’ theory of wages that high wages cause unemployment.” Discuss.
  12. Why are the representatives of organized labour as well as of employers in this country opposed to compulsory arbitration as a method of settling trade disputes?

 

Tuesday, May 31, 1932. 9 — 12.
GENERAL PRINCIPLES II.

A.

  1. How is elasticity of demand measured? Under what conditions is the demand for some product likely to be very elastic?
  2. What evidence would you require in order to discover whether this country is over-populated?
  3. “Price is equal to marginal cost.” Explain the meaning of the word “marginal” in Economics. What is the relation of marginal to average cost?
  4. What are the chief assumptions made in the construction of the competitive theory of value? Are they closely related to the actual conditions of modern economic life?
  5. Show, with a diagram if possible, how a publisher would determine the price that he should charge for a new book.
  6. “Owing to the falling off of sales in 1931, and the consequent increase of overhead cost per unit of output, it has proved necessary for us to advance the prices of our products.” Examine the validity of this argument as applied to a short period.

B.

  1. By what means can a central bank control the level of prices in a country?
  2. What effect on prices would you expect (a) if the practice of paying wages by cheque became more common, (b) if wages were paid monthly instead of weekly?
  3. What is meant by the “purchasing power parity” theory of exchanges? How far do you consider it possible to calculate the present equilibrium rate of exchange between the pound and the dollar?
  4. Can two countries both gain by the existence of a trade between them?
  5. Explain in detail how a Bill of Exchange serves to make payments between persons in different countries.
  6. Give some account of the legal enactments at present governing the issue of paper money in this country, and of the more important changes which have been made in the last hundred years.

 

Tuesday, May 31, 1932. 1½ — 4½.
ESSAY SUBJECTS.

Write an essay on one of the following subjects:

  1. Mr MacQuedy: “Then, sir, I presume you set no value on the right principles of rent, profit, wages and currency?”
    The Rev. Dr. Folliott: “My principles, sir, in these things are, to take as much as I can get, and to pay no more than I can help. These are every man’s principles, whether they be the right principles or not. There, sir, is political economy in a nutshell.”
    (T. L. Peacock: Crotchet Castle.)
  2. The future of party government.
  3. The British Empire as an economic unit.
  4. High Finance.
  5. Sweepstakes.

 

Wednesday, June 1, 1932. 9 — 12.
ENGLISH ECONOMIC HISTORY

  1. Give some account of the “old colonial system” and estimate its importance in the economic development of England from 1660 to 1776.
  2. Examine the causes and the principal consequences of the enclosure movement of the eighteenth and early nineteenth centuries.
  3. “So far from originating cruelty to children, the factory system called attention to the evil by concentrating it where all could see.” Discuss this view.
  4. Compare the outlook and objectives of the Chartists with those of the advocates of the Repeal of the Corn Laws.
  5. What were the principal reasons for the industrial leadership of Great Britain in the ‘fifties and ‘sixties of last century?
  6. “All through the nineteenth century the railways have been the great factor making for the extension of the sphere of State action and the abandonment of the idea of free competition.” Discuss.
  7. Outline the development of direct taxation from Pitt to Gladstone.
  8. Describe and account for the main changes that took place in the general level of gold prices between 1850 and 1914.
  9. What were the principal changes in agricultural organization and policy from the acute depression of the ‘seventies and ‘eighties to the outbreak of the World War?
  10. Give some account of the growth of Trade Unionism and the Labour Movement between 1867 and 1913.
  11. Why did the Protectionist campaign of Mr. Joseph Chamberlain fail, and that of Mr Neville Chamberlain succeed?

 

Wednesday, June 1, 1932. 1½ — 4½.
ECONOMIC STRUCTURE

  1. Give some account of the technical advantages of large scale production in any one industry with whose conditions you are acquainted.
  2. What is meant by “the external economies” of an industry? What difficulties are involved in this conception?
  3. How would you account for the fact that some industries are much more localized than others? In what ways do you consider it likely that the development of the cheaper transmission of electricity will alter the location of British Industries?
  4. Give some account of the objects and methods of Scientific Management.
  5. If in any firm the most efficient technical size of several processes were too large for efficient management, how could the organization be adjusted to minimize this loss of efficiency?
  6. What are the proper functions of advertisement? Do you consider that the total expenditure on advertising in this country is excessive?
  7. How would you explain the fact that in many districts almost every firm turns out more than one product, instead of specializing on a single product?
  8. Describe the more important channels whereby saving is placed at the disposal of industry. What suggestions would you make for their improvement?
  9. Which of the various arguments used to justify the imposition of tariffs at the present moment do you consider valid?
  10. Give an account of the monopoly organisations in any one important British Industry. What factors limit the power of the monopolist to raise prices in the case that you are considering?
  11. Describe the methods employed for the marketing of either cotton or wool at each stage from the raw material to the finished product. Explain why each particular method is adopted, and consider whether any change in the method is desirable.

 

PART II.

Monday, May 30, 19932. 1½ — 4½.
SUBJECTS FOR AN ESSAY.
(OLD AND NEW REGULATIONS.)

  1. Thrift.
  2. The Future of the Party System.
  3. Sky-writing.
  4. The Scope and Method of Realistic Economics.
  5. Disarmament as a Practical Problem.
  6. Man in the Machine Age.

 

Monday, May 30, 1932. 9—12.
MONEY, CREDIT AND PRICES.
(OLD REGULATIONS.)

  1. Do you consider that the legal regulation of the Central Bank’s minimum gold reserve serves a useful purpose in a gold standard country? What form of regulation, if any, do you recommend?
  2. Are there good reasons for holding that the “rate of saving” and “rate of investment” should appear as terms in the equation used to demonstrate the principal factors which govern the value of money?
  3. Explain the working of a forward foreign exchange market. How far can it obviate the inconveniences which arise when there is no fixed par of exchange between currencies?
  4. If it had been decided to stabilize the purchasing power of the national standard of value in terms of an index number of prices, what main categories of goods and/or services would you include in the index number?
  5. What considerations have to be taken into account in analyzing the causes which determine the velocity of circulation of money?
  6. If the principal nations of the world decide to adhere to a gold standard in future, would you consider that the impeding gold shortage gives ground for alarm? State your reasons.
  7. In what respects, if any, do you conceive that the policy of the Federal Reserve System has been open to criticism in the period from 1926 to the present day?
  8. “The resistance of wage earners to reductions in money wages has been of value in saving us from the worst excesses of deflation.” Examine this contention.
  9. On what lines should the Indian currency system be developed?
  10. Do you agree with the view that public works designed to stimulate employment in a slump “are a mere piece of ritual, achieving nothing which could not equally well be achieved b the banking system acting alone, through a sufficiently great alteration in tis terms of lending”?
  11. Would the formation of a large “sterling area” be of advantage or disadvantage to this country, in your opinion, if she were endeavouring to maintain a stable currency divorced from gold?

 

Monday, May 30, 1932. 9—12.
POLITICAL THEORY.
(OLD REGULATIONS.)

 

  1. “The State may be defined as a juridically organized nation.” Discuss this definition.
  2. Would you agree that political obligation is equally binding on the citizen whether he lives in a democracy or under a dictatorship?
  3. Dicey contended that democracy and “collectivism” were inconsistent. Do you accept his contention?
  4. Would you consider that the conception of a right of property as belonging to the individual has permanent value, or would you regard it as only characteristic of a particular phase of social development?
  5. In what sense can we speak of law as being “enacted,” and what are the organs of such enactment in the modern State?
  6. How would you analyse the conception of “public opinion,” and by what methods would you suggest that such opinion should be brought to bear on political government?
  7. On what principles, and by what methods, would you impose limits (if any) on liberty of the expression of thought?
  8. Discuss the prison as one of the institutions of political life, with reference to (a) the purpose which it should serve, and (b) the methods which it should employ.
  9. It has been said that “the process of discovering the Sovereign is in all modern States the same.” By what process would you seek to discover the Sovereign in the modern State?
  10. “The State properly intervenes not to conduct the economic business of the country, but to uphold social standards.” Discuss the value of the distinction here suggested.
  11. ”The problem of our days is not the Man versus the State, as it was when Herbert Spencer wrote in 1884, but the State versus the Group.” Comment on this statement, explaining the sense which you would attach to the term “Group.”

 

Tuesday, May 31, 1932.  1½ — 4½.
DISTRIBUTION AND LABOUR.
(OLD REGULATIONS.)

  1. “Every factor of production tends to be remunerated at a rate equivalent to its marginal net product of commodities in general.” Does this statement need any modifications or qualifications when it is applied to land which are not equally necessary when it is applied to the other factors? Give your reasons.
  2. “The Dole has kept up wages above their proper level.” What are the arguments in support of this view? Discuss their soundness. What changes in public policy, if any, do your conclusions suggest as desirable?
  3. What principal factors determine the magnitude of the change which a fall in the general level of commodity prices occasions in the proportion of the national income accruing to the owners of fixed-interest bearing investments?
  4. With the object of creating employment, a grant is made by the exchequer to a local authority for a road-widening scheme. The local authority sets about doing the work as efficiently as possible with the aid of steam navies and other labour-saving machinery. Ought the Exchequer to challenge this actions? Argue the case for and against.
  5. “The money now paid in unemployment benefits should be used to provide employment.” Suggest possible schemes and discuss their soundness.
  6. Compare the relative levels of wages in skilled and unskilled occupations in this country to-day and before the War. Discuss the bearing of any changes upon (a) the supply of skilled workers, (b) industrial contentment, (c) labour organization and policy.
  7. What are the principal differences between the English and German provisions for State action in the regulation of wages and the settlement of industrial disputes? Discuss whether England could with advantage follow the German example in these matters more closely.
  8. Discuss the relative merits of systems under which wage-rates vary with (a) the market price of the product, (b) a cost of living index, (c) a wholesale price index, (d) the profit of the undertaking.
  9. Explain and discuss the place and function of the Trades Union Congress and the General Council of the Trades Union Congress in labour organization.
  10. If you had to measure the change in the general level of money wages in this country since 1900, what difficulties, theoretical and practical, would you encounter?
  11. If a British lace manufacturer proposed to run his establishment continuously on a three-shift system, what procedure would he have to adopt (a) to conform with legal requirements and (b) to meet possible opposition from any quarter?
  12. What are the functions of the Public Assistance Committees? Do you consider that any of them should be transferred to other authorities or modified in any way? State your reasons.

 

Tuesday, May 31, 1932. 1½ — 4½.
STRUCTURE AND METHODS OF GOVERNMENT IN THE MODERN WORLD.
(OLD REGULATIONS.)

  1. Discuss the powers of the Supreme Court, under the American system of government, in disallowing legislation. Would it, in your view, be possible or desirable to institute a judicial body with similar powers in a nonfederal State?
  2. Discuss the methods by which the House of Commons controls finance. Would you regard those methods as adequate, at the present time, for the purpose of regulating the distribution of expenditure or of enforcing general economy?
  3. Describe the effect of the main changes which have taken place in the relations of the component parts of the British Commonwealth between the outbreak of war in 1914 and the passing of the Statute of Westminster in 1931.
  4. Describe the position and powers of committees of the legislature under the French system of parliamentary government. In what ways do they differentiate that system from the system in Great Britain?
  5. What contrast would you draw between the British party system and that of Germany in regard to (a) their influence on the electorate, and (b) their effects on general working of the Constitution?
  6. How far has England developed the French system of “administrative justice” during the present century? What are the defects of administrative jurisdiction as it exists in England to-day?
  7. What would you regard as the main reforms effected in English local government since 1902, and what further reforms would you advocate in regard to the areas or functions of local authorities?
  8. Describe in general terms the composition and functions of the National Economic Council of Germany, and discuss the tendencies in Great Britain towards the development of a similar system since 1918.
  9. Give some account either of the merits and disadvantages of the system of “indirect rule” in the British Colonial Empire or of the varieties and the working of the system of British Mandated Territories.
  10. Would you agree that there is an increasing recognition in Great Britain of the part which the “Expert” should play in the work of government?
  11. Compare the position of the American Senate with that of the German Reichsrath. How would you account for the comparative weakness of the latter?

 

Wednesday, June 1, 1932. 9 — 12.
STRUCTURE AND PROBLEMS OF INDUSTRY.
(OLD REGULATIONS.)

  1. “The entrepreneur has passed away with the Victorian era. His place has been taken by a combination of financier and salaried manager.” Discuss the truth of this contention.
  2. Discuss the possibility of defining “an industry” so as to make it a useful conception in the analysis of economic structure without doing violence to its popular meaning.
  3. Are there any special circumstances in England to-day that make the rational planning of each industry of more importance than in the past? Illustrate what is included under planning from your knowledge of any one industry.
  4. Explain the nature and causes of any differences that occur in the marketing of (a) producers’ goods, (b) consumers’ goods, and the effect of these differences upon the location of the plants making these two types of goods.
  5. How far can statistical proof be adduced for the contention that some manufacturing industries are subject to the law of increasing returns?
  6. What precisely are the supposed defects in a system of private enterprise that have led to schemes for the public control of such activities as transport, generation of electricity and the supply of water?
  7. “Co-operative societies in agriculture are nothing more nor less than cartels, and threaten the consumer with the same sort of exploitation.” Comment.
  8. What forms of integration and combination seem to you to best calculated to reduce the risks of industrial fluctuations, and why?
  9. Discuss the effect on national and international localization of industry of the modern and the possible future development of (a) the telephone, (b) road transport, (c) commercial aviation.
  10. Are we bound to expect a contraction in British industries manufacturing for export? What statistical sources and methods would you use to estimate the extent of such contraction, and to indicate industries and services to which investment might advantageously be directed or transferred?
  11. Discuss any differences between British and foreign systems of education and training in respect of their effect upon the costs of industrial leadership at home and abroad.

 

Tuesday, May 31, 1932. 9 — 12.
ECONOMIC PRINCIPLES.
(OLD AND NEW REGULATIONS.)

  1. Use “the famous fiction of the ‘stationary State’” to illustrate the uses and abuses of abstraction in the treatment of economic problems.
  2. Analyse, and illustrate by examples, the various ways in which a change in the supply of one commodity may affect the demand for others.
  3. “Under conditions of simple competition in a perfect market, the price of a commodity must, in the long run, be equal both to the marginal and to the average cost of producing it in a representative firm.”
    Explain precisely, what you take this statement to mean, and briefly discuss its validity.
  4. What effect might a substantial tax upon imports of wheat and wheaten flour be expected to have, in this country, upon (a) the price of bread, (b) farming profits, (c) agricultural rents? Give reasons for your answer.
  5. What functions does a company promoter perform, and how are his earnings determined? Frame your answer so as to show how it fits into your general theory of distribution.
  6. In a country like Great Britain, how would an increase in the supply of capital affect the earnings of labour? What do you understand by “an increase in the supply of capital”?
  7. In what circumstances will a monopolist charge different customers different prices for the same product?
  8. How would you proceed if you were required to decide whether profits in any given industry were ‘normal”?
  9. For what reasons may a country permanently import part of its supply of a commodity and produce part at home; and what determines, in such a case, the proportion of the total supply which is imported?
  10. When several different things are produced by the same firm, on what economic principle, if any, is it possible to assign a separate cost of production to each of them?
  11. If the habit of keeping a banking account were to spread among wage-earners in this country, how would the general level of prices be affected?

 

Wednesday, June 1, 1932. 9 — 12.
INTERNATIONAL LAW.
(OLD AND NEW REGULATIONS.)

  1. Discuss the modern theories of the basis of obligation in International Law.
  2. What is the present position in International Law of the right of navigation of international rivers?
  3. What are the sanctions provided by the Covenant of the League of Nations for its enforcement? They have been termed “Pseudo-Sanctions”; do you consider there is any justification for this?
  4. Under what circumstances is a war-ship justified in interfering with a merchant ship of another State on the high seas in time of peace?
  5. What is the connection between the Permanent Court of International Justice and the League of Nations?
    What is implied by signing the “Optional Clause”?
  6. Explain the circumstances under which a State may (a) sue, (b) be sued in a foreign Court of Law.
  7. Article 21 of the Covenant of the League of Nations speaks of “regional undertakings like the Monroe Doctrine for securing the maintenance of peace.”
    Comment on this.
  8. Give the provisions of the Kellogg-Briand Pact, 1928. What methods exist for the pacific settlement of international disputes?
  9. What is the position of
    (a) Sea-borne Mails,
    (b) Submarine Cables,
    in time of war?
  10. What were the methods employed by the Allied Powers in the restriction of enemy commerce during the war of 1914-18? Discuss their legality under the rules of International Law

 

Wednesday, June 1, 1932. 1½ — 4½.
PUBLIC FINANCE.
(OLD AND NEW REGULATIONS.)

  1. What considerations of equity are involved by the action of a Government which reduces rewards to its employees either (a) as a measure to repair a national deficit, or (b) as an example to other employers?
  2. Consider whether there are any grounds for believing that taxation should be progressive if it is to impose equal sacrifice.
  3. “Twenty-five years ago differential taxation was first levied on unearned as against earned income. The time has now come to differentiate between unearned income from fixed-interest securities and that from fluctuating sources.” Is this proposed differentiation (a) desirable (b) feasible?
  4. In what sense, if any can (a) the Income Tax, and 8b) Local Rates be said to enter into the cost of production of goods?
  5. Discuss the possibility of devising a scheme for the taxation of incremental site values which does not place an unfair burden on existing landowners as a class compared with other property owners.
  6. Examine the view that Family Endowment has a good claim to be made a charge on the Exchequer.
  7. If a Board, having a monopoly of the import and export trade of a country and able to buy and sell at current prices at home and abroad, were instructed to maximize its profits, would the effect be more or less advantageous to the country than the flow of trade in free trade conditions?
  8. Compare the economic effects of devaluating the currency of a country by x% in terms of all foreign currencies, with those of imposing an x% ad valorem tariff on all her imports together with an x% ad valorem bounty on all her exports.
  9. What fiscal policy would you adopt if your sole object were to secure the reduction of foreign import duties affecting British exports?
  10. Compare the proper policy of the central authority with that of local authorities with regard to the expansion or contraction in the volume of their outstanding indebtedness.
  11. On what principles should a municipality owning a tramway system proceed in computing the fares to be charged to its passengers?

 

Thursday, June 2, 1932. 1½ — 4½.
THE ECONOMIC DEVELOPMENT OF THE UNITED STATES.

  1. Down to 1860 the economic relations of Great Britain and the United states were complementary rather than rival: after 1870 they were sharply rival.” Comment.
  2. Give examples of the influence of English thought and practice upon the labour movements of America in the early part of the nineteenth century.
  3. Why did the American Mercantile Marine fall away after the Civil War?
  4. Can it be argued that the North and South went to war over any other issue than slavery?
  5. How far was the Tariff responsible for the growth of Trusts? Test your conclusion by reference to particular Trusts.
  6. Distinguish between the “Old” and the “New” Immigration, in respect of (a) its origin, (b) its contribution to American economic life.
  7. Discuss the effect of the disappearance of free land on the economic and social structure of the United States.
  8. Show the significance of the concept of an economic metropolis in an examination of the distribution and range of American industry and commerce.
  9. Indicate, by a sketch map if possible, the regional specialization of American agriculture.
  10. Compare the experience of the United States under the paper dollar of 1862-79 with the experience of England under the paper pound of 1797-1821.

 

Monday, May 30, 1932. 9 — 12.
MONEY.
(NEW REGULATIONS.)

  1. Do you consider that the effects of Peel’s Bank Act were salutary?
  2. Are there good reasons for holding that the “rate of saving” and “rate of investment” should appear as terms in the equation used to demonstrate the principal factors which govern the value of money?
  3. Explain the working of a forward foreign exchange market. How far can it obviate the inconveniences which arise when there is no fixed par of exchange between currencies?
  4. If it had been decided to stabilize the purchasing power of the national standard of value in terms of an index number of prices, what main categories of goods and/or services would you include in the index number?
  5. What considerations have to be taken into account in analyzing the causes which determine the velocity of circulation of money?
  6. Would you have been a mono-metalist or a bi-metalist in the year 1886? State your reasons.
  7. In what respects, if any, do you conceive that the policy of the Federal Reserve System has been open to criticism in the period from 1926 to the present day?
  8. “The resistance of wage earners to reductions in money wages has been of value in saving us from the worst excesses of deflation.” Examine this contention.
  9. “A series of accidents gave India a more satisfactory standard of value than England had in the period from 1873 to 1923.” Discuss.
  10. Do you agree with the view that public works designed to stimulate employment in a slump “are a mere piece of ritual, achieving nothing which could not equally well be achieved by the banking system acting alone, through a sufficiently great alteration in its terms of lending”?
  11. Would the formation of a large “sterling area” be of advantage or disadvantage to this country, in your opinion, if she were endeavouring to maintain a stable currency divorced from gold?

 

Tuesday, May 31, 1932. 1½ — 4½.
LABOUR.
(NEW REGULATIONS.)

  1. Sketch in outline the course followed by real wages in this country during the second half of the nineteenth century, and give some account of the principal causes which brought the changes about.
  2. “The Dole has kept up wages above their proper level.” What are the arguments in support of this view? Discuss their soundness. What changes in public policy, if any, do you conclusions suggest as desirable?
  3. What principal factors determine the magnitude of the change which a fall in the general level of commodity prices occasions in the proportion of the national income accruing to the owners of fixed-interest bearing investments?
  4. With the object of creating employment, a grant is made by the Exchequer to a local authority for a road-widening scheme. The local authority sets about doing the work as efficiently as possible with the aid of steam navies and other labour-saving machinery. Ought the Exchequer to challenge this action? Argue the case for and against.
  5. “The money now paid in unemployment benefit should be used to provide employment.” Suggest possible schemes and discuss their soundness.
  6. Compare the relative levels of wages in skilled and unskilled occupations in this country to-day and before the War. Discuss the bearing of any changes upon (a) the supply of skilled workers, (b) industrial contentment, (c) labour organization and policy.
  7. What are the principal differences between the English and German provisions for State action in the regulation of wages and the settlement of industrial disputes? Discuss whether England could with advantage follow the German example in these matters more closely.
  8. Discuss the relative merits of systems under which wage-rates vary with (a) the market price of the product, (b) a cost of living index, (c) a wholesale price index (d) the profit of the undertaking.
  9. Trace the history of the “One Big Union” idea and movement in this country since 1800.
  10. If you had to measure the change in the general level of money wages in this country since 1900, what difficulties, theoretical and practical, would you encounter?
  11. What were the principal influences determining the development of factory legislation in this country between 1825 and 1878?
  12. What are the functions of the Public Assistance Committees? Do you consider that any of them should be transferred to other authorities or modified in any way? State your reasons.

 

Wednesday, June 1, 1932. 9 — 12.
INDUSTRY.
(NEW REGULATIONS.)

  1. “The entrepreneur has passed away with the Victorian era. His place has been taken by a combination of financier and salaried manager.” Discuss the truth of this contention.
  2. Discuss the possibility of defining “an industry” so as to make it a useful conception in the analysis of economic structure without doing violence to its popular meaning.
  3. Are there any special circumstances in England to-day that make the rational planning of each industry of more importance than in the past? Illustrate what is included under planning from your knowledge of any one industry.
  4. Explain the nature and causes of any differences that occur in the marketing of (a) producers’ goods, (b) consumers’ goods, and the effect of these differences upon the location of the plants making these two types of goods.
  5. How far can statistical proof be adduced for the contention that some manufacturing industries are subject to the law of increasing returns?
  6. Consider the reasons for the introduction and development of municipal trading during the nineteenth century.
  7. How far is it true that since the days of Malthus science and invention have entirely offset any tendency to diminishing returns in the various branches of agriculture?
  8. What forms of integration and combination seem to you to best calculated to reduce the risks of industrial fluctuations, and why?
  9. Discuss the effect on national and international localization of industry of the modern and the possible future development of (a) the telephone, (b) road transport, (c) commercial aviation.
  10. Are we bound to expect a contraction in British industries manufacturing for export? What statistical sources and methods would you use to estimate the extent of such contraction, and to indicate industries and services to which investment might advantageously be directed or transferred?
  11. Discuss any differences between British and foreign systems of education and training in respect of their effect upon the costs of industrial leadership at home and abroad.

 

Thursday, June 2, 1932. 9 — 12.
THEORY OF STATISTICS (OLD REGULATIONS.)
STATISTICS. (NEW REGULATIONS.)

  1. Give an account of the principal properties of the association table, and tests for or measures of association,. Use Table A to illustrate your remark.
TABLE A.
Nervous Symptoms
Women
engaged in
None or slight Marked Total
Factory work 53 30 83
Clerical work 31 7 38
Total 84 37 121
  1. Explain what is meant by a “frequency distribution” and state how, given the data, you would proceed to compile the distribution, directing attention to common faults of presentation. You can use Table B as an illustration. Comment on the dictum “Think in terms of frequency distributions, not of averages.”
TABLE B.
Salaries per day of 10 hours: workers on the French railways, 1896.
Salary Number of workers per 10000
0.25—0.75 10
0.76—1.25 53
1.26—1.75 46
1.76—2.25 145
2.26—2.75 1010
2.76—3.50 3545
3.51—4.25 1921
4.26—5.50 2009
5.51—6.50 702
6.51—7.50 353
7.51—9.00 150
9.10—11.00 37
11.10—15.00 15
15.10—20.00 4
Total 10000

Find some form of average and measure of dispersion for Table B, giving reasons for your choice.

  1. Prove the formula

{\sigma ^2} = {s^2} + {d^2}
where σ is the standard deviation, is the root-mean-square deviation when deviations are measured from an arbitrary origin, and is the difference of the mean from the arbitrary origin.
Find the standard deviation of the data in Table C, and check your own work by any method you consider most effective.

TABLE C.
Deaths from Glanders, persons England and Wales.
1882 3 1892 5
1883 2 1893 6
1884 2 1894 2
1885 5 1895 3
1886 5 1896 1
1887 6 1897 6
1888 2 1898 4
1889 8 1899 5
1890 3 1900 2
1891 4 1901 4
  1. Table D shows the correlation between the class obtained in Part I and the class obtained in Part II by 500 candidates who sat for both parts of the Natural Sciences Tripos in 1922-30. Find the correlation, and the regression equation for class in Part II on class in Part I, and check the fit of the line of regression. The following are the means and standard deviations: suffix 1 refers to Part I and 2 to Part II.
M1= 1.66 σ12 = 0.4337 σ1 = 0.659
M2= 1.93 σ22 = 0.5445 σ2 = 0.738.

 

TABLE D.
Class in Part II
Class in Part II 1 2 3 Total
1 121 35 156
2 92 117 16 225
3 11 72 36 119
Total 224 224 52 500
  1. Explain what is meant by a “weighted” mean, giving illustrations of its use in connection with index numbers of prices.
    Show that ,
    {M_w} = M + r{\sigma _x}\frac{{{\sigma _w}}}{{\bar w}}
    where Mis the weighted mean of X, the arithmetic mean of X, σthe standard deviation of X, σthe standard deviation of the weights w, {{\bar w}} the mean of the weights and the correlation between X and w. In the light of this formula comment on the statement “Weighting usually has little effect.”
  2. What is meant by the “standard error” of a statistical constant? State carefully the conditions assumed, and explain how these limit the value of the standard error as a general measure of “trustworthiness.”
    Prove the formula for the standard error of the arithmetic mean, and find the standard error of the difference between the means M1, Mbelow, σand σbeing the respective standard deviations and N1, Nthe numbers of observations.
M1= 41.3 σ1= 3.8 N1= 100
M2= 38.9 σ2= 3.2 N2= 64.

 

  1. (1) Test the significance of the association in Table A.
    (2) Test whether the data of Table C show anything but mere fluctuations of sampling.
  2. Table E shows the numbers of married women in England and Wales at ages between 15 and 45, reduced to a total of 1000. By some method of interpolation break up the two final decennial groups into quinquennial groups.
TABLE E
Age Married Women
per 1000
15— 7
20— 100
25— 431
35—45 462
1000
  1. Table F shows the beginning of a life-table for Females (no. 7, 1901-10). Explain the meaning of the columns and calculate the figures that should be inserted in the spaces numbered (1), (2) etc. to (9).
    Find also the life-table death-rate and explain why this greatly exceeds the mean crude death-rate for Females 1901-10, viz. 14.4. What light does this phenomenon throw on the probable future course of the crude death-rate?
TABLE F
Age Age
x lx dx px qx Lx Tx x X
0 1000 1174 .8826 .1174 9163 523820 (7) 0
1 8826 (1) (2) (3) (4) (5) (8) 1
2 8494 (6) (9) 2

 

THURSDAY, JUNE 2, 1932. 1½ — 4½.
PRINCIPLES OF POPULATION.
(NEW REGULATIONS.)

  1. Discuss the main features of Hegel’s conception of “economic” or “bourgeois” Society (die bürgerliche Gesellschaft). How far can that conception be regarded as the basis of Marxian theory?
  2. “The expansion of England…was an expansion of Society, and not of the State” (Unwin). Examine the part played by social co-operation, as distinct from political organization, in the development of modern England.
  3. What is the value, and what are the dangers, of the application of biological metaphors (e.g. “the body politic” or “the social organism”) to the study of political theory?
  4. “A true conception of personality and its claims is the first necessity of political theory.” Discuss this statement.
  5. “Liberty is rightly preferred to equality, when the two are in conflict.” Would you agree with this proposition?
  6. What, in your view, is the final basis of the authority of law?
  7. How far does the idea of a “social contract” afford a satisfactory answer to the problem of political obligation?
  8. Does the conception of “the sovereignty of the national State” necessarily involve the unlimited supremacy of such a State?
  9. How far is the conception of “natural rights” a necessary condition of legal progress?
  10. Is the preservation of a distinction between the different “functions” or “powers” of government essential to the liberty of the subject?
  11. What are the political principles implied in the development of “social services” in England during the present century? What limits would you assign on grounds of principle, to the further extension of such services?
  12. “Democracy is not a particular form of State, but the necessary mode of action of all forms of State.” How far would you accept, or on what grounds would you criticize, this proposition?

 

Source:  Cambridge University. Economics Tripos Papers 1931-1933. Cambridge, UK: University Press, 1933, pp. 28-52.

Image Source: Cambridge University, St. John’s Library from website Vintage Postcards.

Categories
Cambridge Chicago Columbia Economic History Economists Germany Harvard NBER Stanford

Chicago. Friedman memo regarding Karl Bode and Moses Abramovitz, 1947

 

In the following 1947 memo from Milton Friedman to T.W. Schultz we can read two talent-scouting reports on potential appointments for the University of Chicago economics department. One candidate, Karl Bode had been vouched for by Allen Wallis, a trusted friend and colleague of Milton Friedman, but we can easily read Friedman’s own less than enthusiastic report on the meager published work examined, certainly compared to Friedman’s glowing report for his friend from Columbia student days, Moses Abramovitz. But comparing the publications listed in the memo, I certainly wouldn’t fault Friedman’s revealed preference for Abramovitz.

Abramovitz went on to have a long and distinguished career at Stanford and Bode left Stanford for government service with his last occupation according to his death certificate “Planning Director, Agency for International Development (A.I.D.)”

Since Karl Bode turned out to have cast a relatively short academic shadow, I have appended some biographical information about him at the end of this post. But for now just the vital dates: Karl Ernst Franz Bode was born November 24, 1912 in Boennien, Germany and he died March 18, 1981 in Arlington, VA.

__________________

Milton Friedman on Bode and Abramovitz

January 10, 1947

[To:] Mr. Schultz, Economics
[From:] Mr. Friedman, Economics
[Re:] Staff appointments

In connection with staff appointments, I thought it might be helpful if I put down on paper for you the information I have on two persons whose names I have casually mentioned: Karl Bode and Moses Abramovitz.

  1. Karl Bode (Assoc. Prof. of Economics, Stanford)

I know about Bode primarily from Allen Wallis. Allen considers him absolutely first-rate in all respects and recommends him very highly.

Bode, who is now in his early thirties, was born in Germany and, though Catholic of Aryan descent, and the holder of a highly-prized governmental fellowship, left Germany almost immediately after Hitler’s accession. He went first to Austria, then to Switzerland, where he took his Ph.D., in 1935, then to England, where he studied at Cambridge and at the London School. Bernard Haley met him while at Cambridge, was highly impressed with him, and induced him to come to Stanford, where he has been since 1937. He has been on leave of absence since early 1945, first with the Tactical Bombing Survey, then with the Allied Military Government in Berlin. He is expected back sometime this summer.

At Stanford, Bode is responsible for American and European Economic History, and, in addition, has taught advanced courses in Economic Theory. His original interest was in International Trade. He has a contract to write a text on Economic History, but I do not know whether on American or European Economic History.

I have obtained a list of his publications, most of which are fragments or reviews. Three of more general interest are:

(a) A. W. Stonier: “A New Approach to the Methodology of the Social Sciences”, Economica, Vol. 4, p. 406-424, Nov., 1937.

(b) “Plan Analysis and process analysis: AER, 33-348-54, June 1943.

(c) “A Note on the Mathematical Coincidence of the instantaneous and the serial multiplier”, Review of Economic Statistics, 26: 221-222, Nov. 1944.

I have read these. They are too slight to permit a reliable and comprehensive judgment about his capacities; but they are sufficient to demonstrate a clear, logical mind.

Allen tells me that Schumpeter, Haberler, Howard Ellis, and of course, the Stanford people all know him and could provide evidence about his abilities.

 

  1. Moses Abramovitz (member of research staff in charge of business cycle unit, National Bureau of Economic Research.)

Abramovitz got his bachelor’s at Harvard, his Ph.D. at Columbia. He has done some part-time teaching of Theory at Columbia. During the war he was with the Office of Strategic Services, where he worked on foreign economic conditions. He was a member of the reparations commission staff at both the Moscow and Paris Conferences.

Abramovitz and I were fellow graduate students at Columbia, and I have known him rather well ever since. I think him extremely capable, with an excellent mind, broad interests, and an extraordinary capacity for forming a sound judgment from conflicting evidence.

His academic and private research background is mostly in Economic Theory and Business Cycles; but the war years gave him a considerable background, and generated a real interest, in foreign economic relations.

Some of his writings are:

Selected Publications:

An Approach to a Price Theory for a Changing Economy, Columbia University Press, 1939.

Monopolistic Selling in a Changing Economy, Q.J.E., Feb., 1938.

Saving vs Investment: Profits vs Prosperity?Supplement on papers relating to the TNEC, Am. Econ. Rev., June, 1942.

Book on Cyclical behavior of inventories completed and scheduled to be published shortly by Nat’l Bureau of Economic Research.

M.F.

ab

* * * * *

PUBLICATIONS OF KARL BODE

A new approach to the methodology of the social sciences. (With A.W. Stonier): Economica, vol. 4, pp. 406-424, November, 1937.

Prosperität und Depression: Zeitschrift für Nationalökonomie, vol. 8, pp. 597-614, December, 1937.

Review of: Plotnik, M.J. Werner Sombart and his type of economics. 1937. American Economic Review, 28: 522-523, September, 1938.

Review of: Sombart, Werner. Weltanschauung, Wissenschaft und Wirtschaft. 1938. Ibid., 28: 766, December, 1938.

The acceptance of defeat in Germany: Journal of abnormal and social psychology, 38: 193-198, April, 1943.

Plan analysis and process analysis: American Economic Review, 33: 348-354, June, 1943.

Review of: Day, C. Economic Development in Europe. 1942:Journal of economic History, 2: 225-227, November, 1942.

Catholics in the postwar world: America, 71: 347-348, July, 1944

Economic aspects of morale in Nazi Germany: Pacific Coast Economic Association: Papers, 1942. pp. 29-34, 1943.

Reflections on a reasonable peace: Thought, 19: 41-48, March, 1944

Review of: Dempsey, B.W. Interest and usury. 1943: Ibid., 18: 756-758, December, 1943.

German reparations and a democratic peace: Thought, 19: 594-606, December, 1944

A note on the mathematical coincidence of the instantaneous and the serial multiplier: Review of Economic Statistics, 26: 221-222, November, 1944.

 

Source:Hoover Institution Archives. Papers of Milton Friedman, Box 79, Folder 1 “University of Chicago, Minutes. Economics Department 1946-1949”.

__________________

Karl F. Bode
AEA 1969 Directory of Members, p. 41.

Bode, Karl F., government; b. Germany, 1912; student, U. Bonn-Germany, 1931-33, U. Vienna-Austria, 1933-34; Ph.D., U. Bern-Switzerland, 1935; Cambridge-England, 1935-37. DOC.DIS. The Concept of Neutral Money, 1935. FIELDS 2abc, 1c, 4a. Chief, Regional Organization & Program Staff, Intl. Cooperation Adm., 1955-60, asst. dep. dir. for planning, 1960-62; chief, Planning Assistance & Research Div., Agy. for Intl. Dev., 1962-67; dir., Research, Evaluation & Information Retrieval, Agy. for Internat. Dev. since 1967. ADDRESS Vietnam Bur., Agy. for Internat. Dev., Dept. State, Washington, DC 20523.

__________________

 Haberler Report of Mises’s Private Seminar

Regular participants of the seminar were several members of the Mont Pelerin Society – notably Hayek, Machlup, the late Alfred Schutz and in the very early days, John V. Van Sickle. Visiting scholars regarded it a great honor to be invited to the seminar – among them Howard S. Ellis (University of California), Ragnar Nurkse (late Professor of Economics in Columbia University, New York) whose untimely death occurred three years ago, Karl Bode (later in Stanford University and now in Washington), Alfred Stonier (now University College in London), and many others. There was Oskar Morgenstern (now Princeton University), the late Karl Schlesinger and Richard Strigl, two of the most brilliant economists of their time…the unforgettable Felix Kaufmann, philosopher of the Social Sciences in the broadest sense including the law and economics – he also wrote a much debated book on the logical foundation of mathematics – who after his emigration in 1938 joined the Faculty of the New School for Social Research in New York where he taught with great success until his premature death twelve years ago.

Source: Mises’s Private Seminar: Reminiscences by Gottfried Haberler. Reprint from The Mont Pelerin Quarterly, Volume III, October 1961, No. 3, page 20f. Posted at the Mises Institute website.

__________________

 From the Preface of Felix Kaufman’s 1936 book

For the critical editing of the manuscript and of the galleys, I wish to thank most heartily a number of friends in various countries, expecially Dr. Karl Bode, presently of St. John’s College, Cambridge and Dr. Alfred Schütz of Vienna. Dr. Bode has also taken upon himself the great labor of preparing both indexes.

Source: Felix Kaufmann. Theory and Method in the Social Sciences. [English translation of Methodenlehre der Sozialwissenschaften. Wien: Julius Springer, 1936.] from Felix Kaufmann’s Theory and Method in the Social Sciences, Robert S. Cohen and Ingeborg K. Helling (eds.). Boston Studies in the Philosophy and  History of Science, 303. Springer: 2014.

__________________

 Reports from The Stanford Daily

The Stanford Daily, Volume 93, Issue 47, 29 April 1938

Several distinguished scholars from other universities will join the Stanford faculty next year…Dr. Karl Franz Bode, formerly on the faculty of St. John’s College, Cambridge University, England, was appointed assistant professor of economics to succeed Dr. Donald M. Erb who was appointed president of the University of Oregon….

 

The Stanford Daily, Volume 100, Issue 02, 23 September 1941, p. 1.

Econ Department Changes Classes… History of Currency Problems, 118, will he given in fall quarter rather than in the spring quarter. It is a five-unit course, taught MTWThF at 11 a.m. in Room 200Q by Karl F. Bode. Economics 1 and 2 are prerequisites….

 

The Stanford Daily, Volume 103, Issue 86, 28 May 1943, p. 1.

Wilbur Names New Faculty Promotions. Promotions and appointments of faculty members for the academic year 1943-1944 were announced yesterday by Chancellor Ray Lyman Wilbur. … Those promoted from assistant professor to associate professor are … Dr. Karl F. Bode, economics….

 

The Stanford Daily, Volume 111, Issue 20, 7 March 1947, p. 3

President Donald B. Tresidder yesterday announced 37 faculty promotions. The promotions include 11 faculty members to full professorships, six to associate professorships, and two to assistant professorships, together with promotion of 18 members of the clinical faculty at the Stanford School of Medicine in San Francisco….

To professorships … Karl F. Bode, in economics…

 

The Stanford Daily, Vol 119, Issue 7, 13 February 1951, p. 1.

Dr. Karl F. Bode, Stanford economics professor on leave for government duty in Germany, has been appointed deputy economic adviser, Office of Economic Affairs, it has been announced by the office of the U.S. High Commissioner for Germany. Dr. Bode will be stationed in Bonn, Germany. He has been acting chief of the program division in the Office of Economic Affairs.

 

Image Source: Karl Bode from the 1939 Standford Quad.

Categories
Cambridge Exam Questions

Cambridge. Economics Tripos Papers, 1931.

 

While Economics in the Rear-view Mirror’s ambition is to be the boutique blog of economics education  in the United States up through the 1960s, from time to time I’ll venture off the North American continent to explore English and German departments, seminars etc.  Earlier I have transcribed and posted the 1891 guide to the Cambridge Moral Tripos that preceded the Economics Tripos but did have a Political Economy component. Also I have transcribed and posted the exams from Oxford’s Philosophy, Politics, and Economics (PPE) program for 1931 that I found in Wesley Clair Mitchell’s papers. As serendipity would have it, my trip last year to the Library of Congress yielded several years’ worth of exams from the Cambridge Economics Tripos. Below you will find the examination papers for 1931 that conveniently coincide with the PPE papers from Oxford. 

I recently found that a copy of the 1931-1933 Cambridge Economics Tripos is available at hathitrust.org.

________________________

ECONOMICS TRIPOS
PART I.

GENERAL PRINCIPLES I.
(NEW REGULATIONS.)

MONDAY, JUNE 1, 1931. 9—12.

  1. Explain clearly the meaning of the term Marginal Utility and its relation to the conception of Consumer’s Surplus.
  2. How far can a theoretical distinction be drawn between the rent of a house and the rent of a field? Is the distinction in all cases valid?
  3. “There is no real connection between price and cost of production throughout an industry as a whole.” Discuss.
  4. In what conditions would competition be said to be perfect? How far and for what reasons is competition in practice often imperfect?
  5. Explain in what circumstances it is possible that a commodity will be produced under conditions of Diminishing Returns. Is it conceivable that the same commodity should be produced at one time under Diminishing, at another under Increasing Returns?
  6. “Russia can grow wheat more cheaply than England because all rents have been abolished in Russia, while rents must still be paid in England.” Examine the validity of this argument.
  7. In what sense can it be said that labour has (a) a demand price, (b) a supply price, (c) a cost of production?
  8. What is likely to be the influence upon the rate of interest of improvements of industrial technique?
  9. Explain, if possible with diagrams, what considerations will influence a monopolist in determining the price that he should charge for his product. In what circumstances will the price be considerably higher than the competitive price?
  10. How would you explain the differences (a) between the wages of coal miners and agricultural workers in this country, (b) between the wages of coal miners in this country and coal miners in Poland?

 

ECONOMIC STRUCTURE.
(NEW REGULATIONS.)

MONDAY, JUNE 1, 1931. 1.30—4.30.

  1. What are the advantages and disadvantages of limited liability?
  2. Explain the factors which determine the size of the productive unit in different industries, giving examples.
  3. What are the functions of a merchant, particularly in the export trades? When is it likely to be to the advantage of a British manufacturer to become his own merchant?
  4. “Left to himself, the consumer would never welcome mass production.” Comment.
  5. Classify the relative importance of the different industries in any neighbourhood with which you are acquainted, explaining how they came there and why they stay there.
  6. In what ways do the problems of management set a limit to the size of the unit of control in industry?
  7. Critically examine the argument for and against vertical combination.
  8. Outline proposals for the reorganization in this country of either shipbuilding or cotton manufacture.
  9. Describe the economic functions of the London Stock Exchange.
  10. Figures published by the Ministry of Labour suggest that industrial employment in Great Britain is moving south-eastwards. Can you account for this?
  11. Under what conditions is an industry likely to become a monopoly? Describe briefly methods for the State control of industrial monopoly.

 

GENERAL PRINCIPLES II.
(NEW REGULATIONS.)

TUESDAY, JUNE 2, 1931. 9—12.

A.

  1. “If an increase in cost of production causes the price of a commodity to rise, the resulting fall in demand will lower its price again and cause it to return eventually to its former level.” Point out the confusion involved in this statement, and illustrate your answer if possible by a diagram.
  2. Consider the nature and importance of the distinction between real wages and nominal wages, and between wage rates and total earnings.
  3. What is likely to be the effect of an increased demand for mutton on the prices of (a) mutton, (b) beef, (c) wool, (d) the wages of woolen workers? Give reasons in each case for your views.
  4. What are the chief characteristics of the English system of land tenure? What do you consider to be the main advantages and disadvantages of this form of tenure?
  5. What causes determine the normal rate of interest in England? What would be the effects of a fall in this rate of interest on the price of (a) War Loan, (b) land, (c) houses, (d) vintage port?
  6. Under what conditions will a check to the supply of a factor of production cause a large increase in its price?

B.

  1. Give a brief account of the modern English banking system, explaining the conditions under which the joint-stock banks can increase their funds in emergency.
  2. What do you understand by the purchasing power of money?
  3. What is meant by the balance of trade? Under what conditions is it possible for a country’s trade returns to show a continuous excess of exports over imports?
  4. What is the difference between a gold standard and a gold-exchange standard? Give examples.
  5. What determines the rate of exchange between the pound sterling and the American dollar?
  6. Is it possible for a country to be undersold all round by its competitors?

 

ESSAY.
(NEW REGULATIONS.)

TUESDAY, JUNE 2, 1931. 1.30—4.30.

Write an essay on one of the following subjects:

  1. “It is a kind of Proverb attending the Character of English Men, that they are better to improve than to invent” (Defoe).
  2. Soviet Farming.
  3. “That outpost of economic empire—the Argentine Republic.”
  4. The United States of Europe.
  5. “Ill fares the land, to hastening ills a prey, where wealth accumulates and men decay.”
  6. England’s Decline and Fall.

 

ENGLISH ECONOMIC HISTORY.
(NEW REGULATIONS.)

WEDNESDAY, JUNE 3, 1931. 9—12.

  1. Give some examples of the way in which economic events have left their impress on economic doctrine.
  2. Contrast the fiscal policy of Gladstone with that of Joseph Chamberlain.
  3. Examine the reasons which led to the localization of the major portion of the woolen industry in the West Riding of Yorkshire.
  4. Estimate the importance of the Chartered Companies for the expansion of England’s foreign trade after 1600.
  5. Illustrate the attitude of Parliament during the nineteenth century towards
    (a) Monopolies, (b) Joint-stock enterprise.
  6. Estimate the services rendered to England by Lord Shaftesbury.
  7. “The industrial revolution was the precursor of a commercial revolution, which was just as important.” Comment.
  8. Illustrate from English history the qualities required from a great inventor.
  9. “War always brings prosperity to agriculture, and Peace, when it comes, depression.” Comment.
  10. Account for the rise of the domestic system in England.
  11. “The repeal of the Corn Laws was the most important political event between the first and second Reform Bills.” Discuss.

 

SOCIAL PROBLEMS.
(NEW REGULATIONS.)

WEDNESDAY, JUNE 3, 1931. 1.30—4.30.

  1. Explain the difficulties involved in any attempt to measure exactly the National Dividend of Great Britain, and to compare it at two dates.
  2. Describe briefly the results of any one investigation into the extent and causes of poverty.
  3. Give some account of the changes in the level of real wages in this country since 1800, with particular attention to changes since 1914.
  4. Consider the importance of the following as causes of unemployment: (1) labour turnover, (2) the need for a reserve of labour, (3) immobility of labour. In what ways can their effects be diminished?
  5. Compare the efficacy of different methods of wage payment as incentives to increase output.
  6. In what conditions do your think it desirable that a government should intervene to fix minimum wages in a trade? What powers of intervention does the Minister of Labour at present possess?
  7. Examine briefly the following methods of dealing with the present unemployment problem: (1) public works, (2) emigration, (3) leaving it to private enterprise.
  8. What means have been suggested for making industry more democratic? How far have they been successful?
  9. Do you consider that the real incomes of workers can be increased more effectively by a rise of wages or by an increased expenditure upon social services?
  10. “The strength of Trade Unionism has been the greatest obstacle to improvements of industrial methods.”
    “The most powerful incentive to improvement has been the encroachment of wages upon the profits of employers.”
    Where does the truth lie?
  11. Give some account of the system of Unemployment Insurance in this country. Subject to what conditions may a man or woman draw benefit? What changes would you suggest in the present regulations?

 

PART II.

ECONOMIC PRINCIPLES.
(OLD AND NEW REGULATIONS.)

MONDAY, JUNE 1, 1931. 9—12.

  1. How far do you consider the theory of Economics still to depend upon the hypothesis of an economic man?
  2. “The theory of rent is nowadays not even of academic interest.” Discuss.
  3. “The mechanism of increasing returns is not to be discerned adequately by observing the effects of variations in the size of an individual firm or of a particular industry, for the progressive division and specialization of industries is an essential part of the process by which increasing returns are realized.” Discuss the truth and significance of this conclusion of Professor Allyn Young.
  4. If an employer finds it necessary to reduce output temporarily, what are the factors he should take into account when deciding whether
    1. to dismiss some of his employees,
    2. to maintain the working-force on short time?
  5. “The doctrine that the earnings of a worker tend to be equal to the net product of his work has by itself no real meaning; since in order to estimate net product we have to take for granted all the expenses of production of the commodity on which he works, other than his own wages.” Comment.
  6. Examine the argument that reductions in wage-rates, since they diminish the purchasing power of important bodies of consumers, can only aggravate a trade depression.
  7. “A wise national policy would seek to maintain an economic return on the vast capital invested in the English railways by attracting back to them a large part of the traffic which in recent years has been diverted to the roads.” Discuss this contention, making plain the economic principles involved.
  8. Examine the arguments for a monetary policy which permits the commodity price-level to fall in proportion to increases in general productivity.
  9. In a world in which no net annual addition is being made to the stock of capital, would you expect the rate of interest to fall to zero?
  10. “The emphasis laid in modern economic treatises on the theory of exchange value is misplaced. The true subject of economics is not the terms on which goods exchange for one another, but the forces which determine the magnitude of a country’s productive resources and the uses to which they are put.” Discuss.
  11. How much truth, if any, is there in the statement that “exports pay for imports”?

 

STRUCTURE AND PROBLEMS OF INDUSTRY.
(OLD REGULATIONS.)

MONDAY, JUNE 1, 1931. 1½ —4½.

  1. What are the main changes in the localisation of manufacturing industry in Great Britain since the War? To what influences do you attribute them?
  2. What difficulties would you meet in attempting to compare costs of production in the same industry in different countries?
  3. In the English cotton industry the typical firm is specialised either to spinning or to weaving; in other countries the typical firm combines spinning and weaving. How do you account for this difference? and do you expect it to persist?
  4. “Rationalisation is merely a new name for monopoly.” “Rationalisation is merely a euphemism for company-promoting.” Criticise these statements, and explain, with reference to some one industry, what you understand by Rationalisation.
  5. British exports to the East of tobacco, oil and fertilisers are distributed by subsidiaries of the producing companies; most other exports are sold by export merchants to merchants at the ports, who in turn leave the internal distribution to native dealers. What are the reasons for this difference of practice? Could the former method be applied with advantage to other exports?
  6. If you were engaged by an American investment trust to advise them on the investment of a portion of their funds in ordinary shares in this country, in what industries, and for what reasons, would you recommend investment?
  7. “Wage-rates in 1929 were approximately at the same level as in 1924, while the cost of living had fallen 5 per cent.; real wages had, therefore, risen about 5 per cent. But the Board of Trade index of industrial production showed an increase in 1929 of 11 per cent. over 1924; therefore there was a case for raising money wages.” Examine this argument.
  8. Explain the operation of either the Federal Farm Board or the Canadian Wheat Pool or the San Paulo Coffee Institute or the British Australian Wool Realisation Association. To what do you attribute its failure or success?
  9. Compare broadly the English and the German attitudes to Restraint of Trade… Explain the functions of the German Kartels Court, and discuss the suitability of such a Court to English conditions.
  10. In what sense, if at all, is there a Science of Management? Can Industrial Administration be taught?
  11. In England Local Authorities are obliged to arrange for the amortization within a definite period of all loans raised for the purpose of financing productive undertakings. Do you regard this as an undesirable handicap on public as compared with private enterprise?

 

STRUCTURE AND METHODS OF GOVERNMENT IN THE MODERN WORLD.
(OLD REGULATIONS.)

MONDAY, JUNE 1, 1931. 1½ —4½.

  1. Describe some of the methods by which central control of local government is provided for in modern States, and discuss their success.
  2. Discuss the view that the Referendum and the Initiative, whatever may be their merits under other systems, are incompatible with Cabinet government.
  3. Compare the means for securing that effective government shall be carried on in an emergency in the United Kingdom and the German Reich respectively.
  4. Compare the system of relationships between politicians and civil servants in England with that in France or any other country.
  5. Explain the unique strength of the United States Senate among contemporary Second Chambers, and discuss the part it plays in American government.
  6. Discuss whether the German Reich should more correctly be described as a federal or a unitary State.
  7. Compare the parts played by parties in the systems of government of the United Kingdom and the United States respectively.
  8. How far may Great Britain be said to fall behind continental countries in providing the individual with legal remedies against State action?
  9. “The constitutional position of the President in the post-war European republics follows the French rather than the American model.” How far is this true, and how do you account for what has occurred?
  10. Illustrate by reference to the respective constitutional positions of the Governor of a Crown Colony, the Viceroy of India, the Governor of an Indian Province, the Governor of an Australian State, and the Governor-General of a Dominion, the development in the office of the “royal governor.”
  11. Illustrate from the constitutions of the British Empire possible methods of safe-guarding the interests of racial or religious minorities (or majorities) where the population is not homogeneous.

 

MONEY, CREDIT AND PRICES.
(OLD REGULATIONS.)

TUESDAY, JUNE 2, 1931. 9—12.

  1. “I am never wary of preaching in the wilderness ‘the only very important thing to be said about currency is that it is not nearly as important as it looks.’” (Marshall.) Do you agree?
  2. Indicate the circumstances in which a Central Bank can most effectively enforce its policy by (a) changes in the bank-rate, (b) open-market operations, (c) rationing of credit.
  3. Has the recent growth of the hire-purchase system for consumption-goods either precipitated or aggravated the slump in the U.S.A.?
  4. “It is even conceivable that the cash-deposits may remain the same, the savings-deposits may remains the same, the volume of monetary transactions may remain the same, and the volume of output may remain the same; and yet the fundamental price-levels may change.” (Keynes.) Is this conceivable? And if so, does it mean that the traditional version of the quantity theory of money is not merely a truism but an error?
  5. In what circumstances would the successful stabilization of the purchasing power of money in terms of an index-number of prices tend to increase or to diminish the fluctuations in the prices of individual commodities?
  6. Factors affecting the general price-level have been distinguished as acting “on the side of money” and “on the side of goods.” Criticize this distinction, or define it so as to avoid ambiguities.
  7. “The belief that the elasticity of demand for currency can ever be different from unity is based on a misunderstanding of what elasticity of demand is.” Discuss.
  8. During 1930 the imports of gold into France have exceeded the world output of gold. What were the causes of this influx?
  9. If central banks fixed their buying and selling prices of gold wider apart, would the diffusion of trade fluctuations from country to country be checked?
  10. How would the internal price-level of this country be affected by a special tax on incomes from foreign investments?
  11. “The joint-stock banks have attracted to their custody a larger volume of the country’s savings than can be economically employed in short-term commercial credits. Some modification in the traditional practice of English deposit banking is therefore required.” Discuss.

 

INTERNATIONAL LAW.
(OLD REGULATIONS.)

TUESDAY, JUNE 2, 1931. 9—12.

  1. Estimate the present international status (if any) of
    1. Canada;
    2. Bavaria;
    3. the State of Virginia;
    4. the Vatican City;
    5. the Territory of the Saar Basin.
  2. Describe the present state of the movement for the codification of International Law (a) as to Peace, (b) as to War and Neutrality. What are in your opinion the chief obstacles in each case?
  3. You are private secretary to a member of Parliament who sits for a fishing constituency bordering on the Moray Firth and who is requested by his constituents to address them upon what they describe as “the invasion of our waters” by Dutch fishing trawlers. He asks you to coach him upon the law (apart from any statutes) relating to fishing by foreign trawlers (a) in territorial waters and (b) in bays, and also (c) upon a proposal to fix by international convention the breadth of all territorial waters at twelve miles. Advise him.
  4. Explain the expression occurring in Article 15 of the Covenant of the League: “a matter which by International Law is solely within the domestic jurisdiction” of one party to the dispute. Give some illustrations of such matters and refer to any judicial discussion of the expression. Is the category of such matters a fixed one?
  5. Discuss the assertion that “the mandate system adopted at the end of the Great War differs in name only from the old-fashioned system of annexation by the victor of the colonies of his defeated enemy.”
  6. Explain the operation of, and the difference between, “national treatment” and “most-favoured nation treatment” stipulated for in commercial treaties. What are the two principal interpretations of the usual “most-favoured-nation treatment” clause?
  7. Discuss the rules of International Law which are relevant to the growth of trading by Governments, including the rules governing the status of State-owned merchant ships in foreign ports.
  8. Describe, with illustrations, the different ways in which the Permanent Court of International Justice may acquire jurisdiction in respect of a dispute. What do you understand by “non-justiciable disputes”?
  9. Discuss the extent to which the conception of Contraband has changed since the beginning of the Great War, and the effect of that change upon the Declaration of Paris of 1856.
  10. A Government White paper published in 1929 concludes an argument with the sentence: “In other words, as between Members of the League, there can be no neutral rights because there can be no neutrals.” Discuss the accuracy of this statement, having regard both to the Covenant and to the Peace Pact of Paris.

 

SUBJECTS FOR AN ESSAY.
(OLD AND NEW REGULATIONS.)

TUESDAY, JUNE 2, 1931. 1½ —4½.

  1. Business Forecasting.
  2. The Decline and Fall of the British Empire.
  3. The Rights of Shareholders.
  4. “Buy British.”
  5. Republicanism.
  6. Arnold Bennett, the interpreter of industrialism.

 

POLITICAL THEORY.
(OLD REGULATIONS.)

WEDNESDAY, JUNE 3, 1931. 9—12.

  1. In what sense, and to what extent, do you regard it as the place of the State to allot their provinces to other associations?
  2. In what circumstances, if any, would you justify the refusal by the individual of a demand on the part of the State for his military service, and on what grounds?
  3. “La liberté est le droit de faire tout ce que les lois permettent.” Montesquieu.
    “By liberty I mean the assurance that every man shall be protected in doing what he believes his duty against the influence of authority and majorities, custom and opinion.” Acton.
    Discuss the adequacy of these definitions.
  4. Wherein do you consider the originality and importance of Rousseau as a political theorist to lie?
  5. Compare and contrast Mazzini’s doctrine of nationality with those of present-day Italian nationalists.
  6. “The citizen should be moulded to suit the form of government under which he lives.” To what extent do you accept this principle of Aristotle’s?
  7. Examine the implications and discuss the validity of the theory of a “right to work or maintenance.”
  8. What circumstances are required, in your opinion, to justify a demand for “national self-determination” on the part of a community?
  9. Consider the arguments for increased public control of the Press in England.
  10. How far, in your opinion, does (a) illiteracy, (b) failure to vote, (c) ecclesiastical influence justify the withholding or withdrawal of the political franchise?

 

DISTRIBUTION AND LABOUR.
(OLD REGULATIONS.)

WEDNESDAY, JUNE 3, 1931. 9—12.

  1. Examine critically the policy of reducing unemployment by settling new industries in districts where older industries appear to be permanently depressed.
  2. Contrast conventional and “scientific” methods of determining “fair” piece-rates. What are the reasons for varying the rate degressively or progressively according to the total output produced?
  3. Discuss the difficulties of demonstrating statistically that wages tend to equality in trades “which are of equal difficulty and disagreeableness, which require equal natural abilities and an equally expensive training.”
  4. “In the past he had often had occasion to speak of ‘insurance popularly miscalled the dole.’ To-day he was afraid that it might be truer to speak of ‘the dole officially miscalled insurance.’” Discuss this view of Sir William Beveridge expressed in his evidence before the Royal Commission on Unemployment Insurance.
  5. Show by reference to specific trades the reasons for which (a) Trade Boards, (b) Joint Industrial (“Whitley”) Councils have been established in some trades and not in others. Can you suggest possible extensions of either to any further trades?
  6. What effect would (a) the growth of trade unionism among women, (b) the repeal of legal restrictions upon women’s employment, be likely to have upon the wages and employment of men?
  7. What measures may any one employer take to reduce labour costs besides reducing wage-rates or substituting machines for men? Is there any statistical evidence of the exact effect of such measures?
  8. What considerations would you take into account in adjudicating on a claim for reduction of wages in an industry in which no return is being earned on ordinary capital and there is over 15 per cent. unemployment?
  9. How far do you consider the marginal theory of distribution a full and adequate explanation of the actual distribution of incomes among persons?
  10. Analyse the problem of regulating wages and hours in the coal-mining industry as illustrated by events since the war.

 

PUBLIC FINANCE.
(OLD AND NEW REGULATIONS.)

WEDNESDAY, JUNE 3, 1931. 1½ —4½.

  1. Examine the view that the budgetary problems of the creditor countries concerned would be lightened rather than aggravated by a general remission of reparations and war-debts.
  2. The Royal Commission on Transport, reporting at the end of 1930, proposed that the present proportion in which the cost of the roads is shared between Local Authorities and the users of mechanical transport should be reversed, the share of the latter being brought up to two-thirds by the payment into the Road fund of the then existing duty on petrol and of that part of the licence duties on motor vehicles which is at present diverted to the Exchequer. Discuss this proposal.
  3. If you were in charge of the finances of the U.S.S.R., on what part, if any, of the capital employed in the various nationalized industries would you require the payment of interest?
  4. Is there any ground for the opinion that taxation imposed for the purpose of paying interest on internal debt is less onerous to the nation than taxation imposed for the purpose of paying the salaries of Government officials?
  5. Examine the view that improvements in the means of transport, such as suburban Tube extensions, should be financed out of confiscatory taxes on the resultant increments in the site value of surrounding land.
  6. If, as Chancellor of the Exchequer, you had £10 millions a year to dispose of, what considerations would you take into account in deciding between the claims for a further subsidy to working-class housing and those of a subsidy to domestic wheat-growing?
  7. “The arguments again the taxation of imported food which were valid in the early years of the twentieth century have one and all ceased to be valid today.” Comment.
  8. “As soon as the services financed by Government come to exceed the minimum requirements of security and order, the whole notion of equity in taxation becomes unreal and meaningless.” Discuss.
  9. How should the maintenance of those unemployed persons who cannot be brought within the framework of a self-supporting insurance scheme be financed?
  10. “The annual repayment of a substantial block of the National Debt is a fetish to which successive Chancellors of the Exchequer pay lip service, but which no sensible person either expects or desires to see carried out.” Comment.
  11. Discuss broadly the special problems of Public Finance which arise in countries with a Federal form of Government, illustrating your answer by reference to any one of the following countries: The United States, Germany, Australia, India.

 

STATISTICS.
(OLD AND NEW REGULATIONS.)

THURSDAY, JUNE 4, 1931. 9—12.

  1. Define the terms median and quartiles, and discuss the uses, advantages and disadvantages of the median as an average and of the semi-interquartile range as a measure of dispersion.
    Find the medians and quartiles of the two series of index-numbers for prices of twenty foodstuffs below, and use the work to illustrate your comments.
1913 1929
1. 58 77
2. 65 91
3. 66 84
4. 70 91
5. 73 95
6. 73 112
7. 67 95
8. 82 143
9. 92 120
10. 98 132
11. 99 142
12. 102 151
13. 105 175
14. 104 157
15. 95 144
16. 40 42
17. 38 31
18. 88 140
19. 44 54
20. 52 93
  1. Sketch the forms of frequency distributions most commonly met with in practice, and name sources from which examples of each can be drawn.
    Find the mean and standare deviation of the distribution below. Earnings were given to a penny.
Earnings Hands with earnings between limits stated
20s. but less than 25s. 6
25s. but less than 30s. 107
30s. but less than 35s. 490
35s. but less than 40s. 228
40s. but less than 45s. 109
45s. but less than 50s. 33
50s. but less than 55s. 14
55s. but less than 60s. 6
60s. but less than 65s. 4
65s. but less than 70s. 2
70s. but less than 75s. 1
Total 1000

 

  1. State the formula for the correlation coefficient and deduce its principal properties.
    The correlation coefficient between the two series of index-numbers given in Question 1 was calculated, and the following figures obtained:
1913 1929
Arbitrary origins used 76 108
Sums of deviations from these origins -9 +9
Sums of squares of deviations from these origins 9131 30,055
Sum of products of deviations +15, 834

Find the coefficient, and the regression equations.

  1. Give any deduction you prefer of the normal curve of errors, and prove its principal properties. Describe the tables of functions of the normal curve that are available in such a volume as Tables for Statisticians and Biometricians, and their uses in solving problems in sampling.
  2. Show that if random samples of n observations are drawn from an indefinitely large record in which the proportion of A’s is p and of not-A’s is q, the standard deviation of the numbers of A’s in the samples is .
    You are given the following data:
    Father skilled worker:—36 boys: 24 classed as intelligent.
    Father unskilled: —64 boys: 32 classed as intelligent.
    Would you say that the difference was significant?
  3. Give a description and critical discussion of the methods available for investigating the relations between two quantities varying with the time.
  4. Argue the case for the use of the geometric mean in connection with index-numbers of wholesale prices, comparing the advantages and disadvantages against other methods. Name any existing index-number for which it is used, and describe the detailed method of construction.
  5. Indicate the difficulties that are met with in obtaining from Census and Registration data reasonably accurate measures of the mortalities of men engaged in different occupations, and describe the technical methods that are used for indicating relative mortalities.
  6. The following are the ages returned at the Census of 1921 by a sample of male Hindus in Madras. By some process of graduation, make an estimate of the true numbers at 30 and 31 years of age.
Age Numbers Age Numbers Age Numbers
20 8579 30 12294 40 10391
21 1077 31 652 41 460
22 3053 32 2058 42 1105
23 1156 33 672 43 436
24 1786 34 892 44 514
25 9291 35 7723 45 5352
26 1946 36 1437 46 957
27 1595 37 870 47 570
28 2709 38 1362 48 919
29 927 39 467 49 395

INDUSTRY.
(NEW REGULATIONS.)

MONDAY, JUNE 1, 1931. 1½ —4½.

  1. What difficulties would you meet in attempting to compare costs of production in the same industry in different countries?
  2. In the English cotton industry the typical firm is specialised either to spinning or to weaving; in other countries the typical firm combines spinning and weaving. How do you account for this difference? and do you expect it to persist?
  3. British exports to the East of tobacco, oil and fertilisers are distributed by subsidiaries of the producing companies; most other exports are sold by export merchants to merchants at the ports, who in turn leave the internal distribution to native dealers. What are the reasons for this difference of practice? Could the former method be applied with advantage to other exports?
  4. “Wage-rates in 1929 were approximately at the same level as in 1924, while the cost of living had fallen 5 per cent.; real wages had, therefore, risen about 5 per cent. But the Board of Trade index of industrial production showed an increase in 1929 of 11 per cent. over 1924; therefore there was a case for raising money wages.” Examine this argument.
  5. Explain the operation of either the Federal Farm Board or the Canadian Wheat Pool or the San Paulo Coffee Institute or the British Australian Wool Realisation Association. To what do you attribute its failure or success?
  6. Compare broadly the English and the German attitudes to Restraint of Trade. Explain the functions of the German Kartels Court, and discuss the suitability of such a Court to English conditions.
  7. In what sense, if at all, is there a Science of Management? Can Industrial Administration be taught?
  8. “The only trades which it seems possible for a joint-stock company to carry on successfully, without an exclusive privilege, are those, of which all the operations are capable of being reduced to what is called a routine, or to such a uniformity of method as admits of little or no variation” (Adam Smith).
    Examine this statement in the light of the subsequent development of the joint-stock company.
  9. What features has the present depression in British industry in common with those of the nineteenth century?
  10. In what directions, and with what results, did England export capital in the half-century before the War?
  11. In England Local Authorities are obliged to arrange for the amortization within a definite period of all loans raised for the purpose of financing productive undertakings. Do you regard this as an undesirable handicap on public as compared with private enterprise?

 

MONEY.
(NEW REGULATIONS.)

TUESDAY, JUNE 2, 1931. 9—12.

  1. “I am never wary of preaching in the wilderness ‘the only very important thing to be said about currency is that it is not nearly as important as it looks.’” (Marshall.) Do you agree?
  2. Indicate the circumstances in which a Central Bank can most effectively enforce its policy by (a) changes in the bank-rate, (b) open-market operations, (c) rationing of credit.
  3. “It is even conceivable that the cash-deposits may remain the same, the savings-deposits may remains the same, the volume of monetary transactions may remain the same, and the volume of output may remain the same; and yet the fundamental price-levels may change.” (Keynes.) Is this conceivable? And if so, does it mean that the traditional version of the quantity theory of money is not merely a truism but an error?
  4. Factors affecting the general price-level have been distinguished as acting “on the side of money” and “on the side of goods.” Criticize this distinction, or define it so as to avoid ambiguities.
  5. “The adoption of Ricardo’s currency plan in 1925, as well as its rejection a hundred years before, point to the same conclusion: the economic ideas of legislators are always a century out of date.” Comment.
  6. Describe the main fluctuations in the value of silver during the last sixty years, and state their causes.
  7. What are the chief changes in the organization of the London money market since the publication of Bagehot’s Lombard Street?
  8. During 1930 the imports of gold into France have exceeded the world output of gold. What were the causes of this influx?
  9. If central banks fixed their buying and selling prices of gold wider apart, would the diffusion of trade fluctuations from country to country be checked?
  10. How would the internal price-level of this country be affected by a special tax on incomes from foreign investments?
  11. “The joint-stock banks have attracted to their custody a larger volume of the country’s savings than can be economically employed in short-term commercial credits. Some modification in the traditional practice of English deposit banking is therefore required.” Discuss.

 

LABOUR.
(NEW REGULATIONS.)

WEDNESDAY, JUNE 3, 1931. 9—12.

  1. Examine critically the policy of reducing unemployment by settling new industries in districts where older industries appear to be permanently depressed.
  2. Contrast conventional and “scientific” methods of determining “fair” piece-rates. What are the reasons for varying the rate degressively or progressively according to the total output produced?
  3. What effect would (a) the growth of trade unionism among women, (b) the repeal of legal restrictions upon women’s employment, be likely to have upon the wages and employment of men?
  4. What measures may any one employer take to reduce labour costs besides reducing wage-rates or substituting machines for men? Is there any statistical evidence of the exact effect of such measures?
  5. What considerations would you take into account in adjudicating on a claim for reduction of wages in an industry in which no return is being earned on ordinary capital and there is over 15 per cent. unemployment?
  6. How far do you consider the marginal theory of distribution a full and adequate explanation of the actual distribution of incomes among persons?
  7. What is the statistical evidence as to changes in the real earnings of the wage-earning classes and their living conditions during the last hundred years? How far do you think this evidence conclusive?
  8. Compare the post-war attitude of governments and governing classes to the relief of poverty with the attitude prevailing in the Victorian era.
  9. Analyse the problem of regulating wages and hours in the coal-mining industry as illustrated by events since the war.
  10. Account for the fluctuations in the emphasis placed by Trade Unions upon the policies of mutual insurance, political action and the strike, respectively, during the last hundred years.

 

Source: Cambridge University. Economics Tripos Papers 1931-1933. Cambridge: 1933, pp. 5-27.

Image Source: King’s College dining hall, Cambridge, from Wikipedia Commons.

Categories
Cambridge Curriculum Suggested Reading

Cambridge. Guide to the Moral Sciences Tripos. James Ward, editor, 1891

 

 

Just learned today that the plural of Tripos is Triposes. But needn’t worry, I will stick to the singular form as in “Moral Sciences Tripos”. For those curious about all the Triposes offered at Cambridge University at the end of the 19th century,  much valuable information is to be found in The Student’s Guide to the University of Cambridge (Fifth edition, rewritten. Cambridge: Deighton, Bell and Co. 1893). Until Alfred Marshall was able to introduce a new Tripos in Economics and Political Science at Cambridge (see Alfred Marshall: The New Cambridge Curriculum in Economics and Associated Branches of Political Science: Its Purpose and Plan, 1903), the Moral Sciences Tripos of Psychology, Philosophy and Political Economy had served as an important breeding ground for Britain’s future economists.

Each of the individual guides for a particular Tripos could be purchased by the students. Below we have the guide written by the psychologist/philosopher, James Ward, for the Moral Sciences. He notes that John Neville Keynes provided suggestions with respect to Political Economy. I have provided links to just over thirty items in the readings lists.

 

________________

MORAL SCIENCES TRIPOS.
[revised edition, 1891]

Edited by
James Ward, Sc.D.
Examiner for the Moral Sciences Tripos and Lecturer
and Assistant Tutor of Trinity College

________________

NOTE.  For the special recommendations relating to Political Economy the Editor is indebted to Dr [John Neville] Keynes, University Lecturer and formerly Fellow of Pembroke College; and for those relating to Politics and Ethics he is indebted to Mr J.S. Mackenzie, Fellow of Trinity College.

________________

The examination for the Moral Sciences Tripos consists of two parts; and begins, as a rule, upon the Monday after the last Sunday but one in May. No student may present himself for both parts in the same year.

The first part consists of two papers on each of the following subjects: Psychology including Ethical Psychology; Logic and Methodology; Political Economy; together with a paper of Essays.

A candidate for honours in this part must be in his fifth term at least, having previously kept four terms; but nine complete terms must not have passed after the first of these four, unless the candidate has obtained honours in some other Tripos, in which case eleven complete terms may have passed.

The names of the candidates who obtain honours are placed in three classes, each class consisting of. one or more divisions arranged in alphabetical order.

The subjects of the second part of the examination fall into two groups:—(A) Metaphysics, Political Philosophy, Ethics—on each of which there is one paper—and (B) the following special subjects, History of Philosophy, Advanced Logic and Methodology, Advanced Psychology and Psychophysics, Advanced Political Economy. There are two papers on each of these special subjects besides an Essay paper containing questions on all the above subjects. Every student must take one, and may not take more than two, of the special subjects; also every student must take the papers on Metaphysics and Ethics except those who select Advanced Political Economy as a special subject: for such students the paper on Political Philosophy is provided as an alternative for Metaphysics.

A candidate for honours in this part must have already obtained honours in Part I. or in some other Tripos: he must also be in his eighth term at least, having previously kept seven terms; but twelve complete terms must not have passed after the first of these seven.

The names of the candidates who pass are placed in three classes arranged in alphabetical order. No candidate will be refused a first class on the ground that he has taken up only one special subject provided that his work reaches the first class standard in the compulsory subjects and his special subject taken together. In the case of every student who is placed in the first class, the class list will shew by some convenient mark (1) the subject or subjects for which he is placed in that class, and (2) in which of those subjects, if in any, he passed with special distinction.

The following schedules of the different subjects, with lists of books recommended for study, was issued by the Special Board for Moral Science on June 17, 1889.

Schedule of the Subjects of Examination in
Part I. of the Moral Sciences Tripos.

I. Psychology.

  1. Standpoint, data, and methods of Psychology. Its fundamental conceptions and hypotheses. Relations of Psychology to Physics, Physiology, and Metaphysics.
  2. General analysis and classification of states of mind. Attention, consciousness, self- consciousness. Elementary psychical facts: impressions, feelings, and movements; retentiveness, arrest, association; appetite and aversion; reflex action, instinct, expression of feeling.
  3. Sensation and perception. Intensity, quality, and complexity of sensations. Physiology of the senses. Activity and passivity of mind. Localisation of sensations. Psychological theories of time and space. Intuition of things.
  4. Images. Imagination, dreaming, hallucination. Flow of ideas. Interaction of impressions and images. Memory, expectation, obliviscence.
  5. Thought. Comparison, abstraction, generalisation: formation of conceptions. Psychology of language. Influence of society upon the individual mind. Judgment. Psychological theories of the categories.
  6. Emotions: their analysis and classification. Higher sources of feeling: aesthetic, intellectual, social and moral. Theories of emotional expression.
  7. Voluntary action; its different determining causes or occasions, and their operation: Pleasure, pain, desire, aversion, and their varieties: will and practical reason: conscience, moral sentiments, moral perception or judgment, moral reasoning. Conflict of motives, deliberation, self- control. The origin of the moral faculty.

List of books recommended on this subject:

Sully, Outlines of Psychology.
Bernstein, The Five Senses of Man.
Bain, The Emotions and the Will.
Ward, Psychology, Article in the Encyclopedia Britannica, ninth edition.

The following books should also be consulted:

Bain, The Senses and the Intellect.
Dewey, Psychology.
Höffding, Psychologie in Umrissen.
Ladd, Elements of Physiological Psychology.
Lotze, Microcosmus, Vol. I.
Spencer, Principles of Psychology [Volume I; Volume II].

II. Logic and Methodology.

  1. Province of Logic, formal and material.
  2. Logical functions of language: names, and their kinds: formation of general notions: definition, division, and classification: predicables and categories: scientific nomenclature and terminology.
  3. The fundamental laws of thought, and their application to logical processes.
  4. Propositions and their import: opposition and conversion of propositions.
  5. Analysis and laws of syllogism.
  6. The nature of the inductive process: ground of induction: connexion between induction and deduction: analogy.
  7. Uniformities of nature, and their combinations: their analysis, and the methods of discovering and proving them: observation and experiment: scientific explanation: the nature and uses of hypothesis: doctrine of chance.
  8. Error, its nature and causes, and the safeguards against it: classification of fallacies.

List of books recommended on this subject:

Whately, Logic.
Keynes, Formal Logic.
Mill, Logic [Volume I; Volume II]
Jevons, Principles of Science.

The following books should also be consulted:

Bacon, Novum Organon.
Drobisch, Neue Darstellung der Logik.
Mill, Examination of Hamilton, Chapters 17 to 24.
Whewell, Novum Organon Renovatum.
Ueberweg, System of Logic.

III. Political Economy.

  1. The fundamental assumptions of Economic Science, the methods employed in it, and the qualifications required in applying its conclusions to practice; its relation to other branches of Social Science.
  2. Production of Wealth.
    Causes which affect or determine

    1. The efficiency of capital and of labour.
    2. The difficulty of obtaining natural agents and raw materials.
    3. The rate of increase of capital and population.
  3. Exchange and Distribution of Wealth.
    Causes which affect or determine

    1. The value of commodities produced at home.
    2. The rent of land.
    3. Profits and wages.
    4. The value of currency.
    5. The value of imported commodities. Monopolies. Gluts and crises. Banking, and the foreign Exchanges.
  4. Governmental Interference in its economic aspects. Communism and Socialism.
    The principles of taxation: the incidence of various taxes: public loans and their results.

List of books recommended on this subject:

Marshall, Economics of Industry.
Walker, The Wages Question, and Land and its Rent.
Mill, Principles of Political Economy, Books III. and V.
Jevons, Money and the Mechanism of Exchange.
Sidgwick, Principles of Political Economy, Introduction and Book III.
Fawcett, Free Trade and Protection.

The following books should also be consulted:

Bagehot, Lombard Street.
Bastable, Foreign Trade [sic, The Theory of International Trade (1887)].
Farrer, Free Trade and Fair Trade.
Giffen, Essays in Finance, Second Series.
Nicholson, Money and Monetary Problems, Part I.
Rae, Contemporary Socialism.
Sidgwick, Principles of Political Economy, Books I. and II.

Schedule of the Subjects of Examination in
Part II. of the Moral Sciences Tripos.

A.

I. (a) Metaphysics.

  1. Knowledge, its analysis and general characteristics: material and formal elements of knowledge; self-consciousness as unifying principle; uniformity and continuity of experience.
  2. Fundamental forms of the object of knowledge: difference, identity; quantity, quality, relation; space and time; unity, number; substance, change, cause, activity and passivity; &c.
  3. Certainty, its nature and grounds : sensitive, intuitive and demonstrative certainty; necessities of thought;’1 inconceivability of the opposite “; verification by experience.
  4. Criteria applicable to special kinds of knowledge: matters of fact and relations of ideas; logical and mathematical axioms; fundamental assumptions of physical science: causality, continuity, conservation of matter and of energy.
  5. Sources and limits of knowledge: Empiricism, Rationalism, Transcendentalism; relativity of knowledge, its various meanings and implications; distinction of phenomena and things per se; the conditioned and the unconditioned, the finite and the infinite.
  6. Coordination of knowledge: mechanical and dynamical theories of matter; evolution; physical and psychical aspects of life; province of teleology; relation of mind and matter; relation of the individual mind to the universe; problem of the external world; Materialism, Idealism, Dualism; relation of theoretical and practical philosophy.

I. (b) Politics.

  1. Definition of State: general relation of the individual to the State and to Society : connexion of Law with Government in modern states : general view of functions of government : grounds and limits of the duty of obedience to government.
  2. Principles of Legislation in the modern state: right of personal security : rights of property: contract and status: family rights : bequest and inheritance : prevention and reparation of wrongs : theory of punishment : governmental rights : grounds and limits of governmental interference beyond the making and enforcement of laws : principles of taxation.
  3. External relations of states : principles of international law and international morality : war, and its justifications : expansion of states, conquest and colonization : relation of more civilized societies to less civilized.
  4. Distribution of the different functions of government in the modern state : legislative, executive, and judicial organs, their mutual relations, and their modes of appointment : relation of the state to other associations of its members : sovereignty: constitutional law and constitutional morality: constitutional rights of private persons : central and local government: federal states; government of dependencies.
  5. A general historical survey of (a) the development of Law and Government, (b) the chief variations in the form and functions of government in European communities, (c) the relations of these variations to other social differences and changes.

II. Ethics.

  1. Analysis of the moral consciousness; moral sentiment, moral perception, moral judgment, moral intuition, moral reasoning: object of moral faculty; voluntary action, motives, intentions, dispositions, habits, character: freedom of will and determination by motives.
  2. The end or ends of rational action, ultimate good: the standard of right and wrong action: moral law: moral obligation: evil, moral and physical: interest and duty: virtue and vice: moral beauty and deformity: happiness and welfare, private and universal: pleasure and pain, qualitative and quantitative comparison of pleasures and pains: perfection, moral and physical, as rational end.
  3. Exposition and classification of particular duties and transgressions, virtues and vices: different types of moral character: principles of social and political justice.
  4. Relation of Ethics to Metaphysics, Psychology, Sociology and Politics.

 

Special Subjects.

III. History of Philosophy.

A special subject in the History of Philosophy will be announced in the Easter Term next but one preceding that in which the examination is to be held. Students will also be required to have a general knowledge of the History of Philosophy.

IV. Advanced Psychology and Psychophysics.

A fuller knowledge will be expected of the subjects included in the schedule for Part I., and of current controversies in connexion with them. Further, a special knowledge will be required (i) of the physiology of the senses and of the central nervous system, (ii) of experimental investigations into the intensity and duration of psychical states, and (iii) of such facts of mental pathology as are of psychological interest. Questions will also be set relating to the philosophic treatment of the relation of Body and Mind as regards both the method and the general theory of psychology.

V. Advanced Logic and Methodology.

Students will be expected to shew a fuller knowledge of the subjects included in the schedule for Part I., and of current controversies in connexion with them, and the examination will also include the following subjects:—Symbolic Logic, Theory of Probabilities, Theory of Scientific Method, Theory of Statistics.

VI. Advanced Political Economy.

Students will be expected to shew a fuller and more critical knowledge of the subjects included in the schedule for Part I. The examination will also include the following subjects; the diagrammatic expression of problems in pure theory with the general principles of the mathematical treatment applicable to such problems: the statistical verification and suggestion of economic uniformities: and a general historical knowledge (a) of the gradual development of the existing forms of property, contract, competition and credit; (b) of the different modes of industrial organization; and (c) of the course and aims of economic legislation at different periods, together with the principles determining the same.

 

Remarks on the above Schedules.

Students will probably find it best to begin with Political Economy and Logic. The undisputed evidence which a large portion of Logic possesses peculiarly adapts it for beginners: and the principles of Political Economy, while they can be grasped with less effort of abstraction than those of Philosophy, also afford greater opportunity of testing the clearness of the student’s apprehension by their application to particular cases.

Accordingly, in the particular suggestions which follow as to the method of study to be adopted in the different departments respectively, we may conveniently take the subjects in the following order: Logic and Methodology, Political Economy, Psychology1, Metaphysics, Politics, Ethics, and History of Philosophy1. Care has been taken to distinguish the recommendations addressed to students who only aim at the more elementary or more general knowledge which will suffice for Part I., from those which relate to the more full and detailed knowledge—either of the subjects themselves or of the history of doctrine relating to them—which is required in Part II.

1To avoid repetition the reading in these subjects for both parts is included under one head.

 

1. Logic and Methodology.

There are important differences in the range of meaning with which the term Logic is used. In its widest signification, it includes two departments of inquiry which may be to some extent studied independently of each other. The first of these,—to which alone the name Logic was formerly applied, and which still, according to some writers, should be regarded as constituting the whole of Logic,—is concerned with reasonings only in so far as their validity can be determined a priori by the aid of laws of thought alone.

This study is often called, for distinction’s sake, ‘Formal Logic;’ on the ground that it is concerned with the form and not with the matter of thought; i.e. not with the characteristics of the particular objects about which the mind thinks and reasons, but with the manner in which, from its very nature, its normal thoughts and reasonings about them are constructed. It is with this branch that the student should commence, familiarising himself with it by the aid of some elementary hand-book, e.g. Jevons’s Elementary Lessons in Logic, or Fowler’s Deductive Logic.

He should then take Keynes’s Formal Logic as his text-book, consulting other works on the subject when he finds them there referred to, and, in particular, working out a good number of the examples and problems that are set.

The latter portion of Jevons’s Lessons or Fowler’s Inductive Logic may serve as an introduction to Mill’s Logic for those who shrink from facing Mill’s two volumes at once. This work has a much wider scope than that of Formal Logic, as above explained; and in fact deals at length with topics that do not so properly belong to Logic— even according to Mill’s own definition of Logic—as to Methodology, or the theory of the intellectual processes by which the truths of the different sciences have been reached in the past, and may be expected to be reached in the future. It should be observed also that even when Mill is apparently discussing the same topics as those discussed by the formal logicians, he will often be found to treat them in quite a different spirit, and from a different point of view.’ A clear apprehension of this difference can only be attained in the course of the study itself: but it is well that the student should be prepared for it at the outset. The greater portion of Jevons’s Principles of Science is devoted to the description and analysis of the methods of the physical sciences, and contains an almost unique collection of interesting and valuable scientific illustrations. Dr Venn’s Empirical Logic, published since the schedule was issued, should be read carefully either along with or after these works by Mill and Jevons. Whewell’s Novum Organon Renovatum should be consulted in connexion with Mill’s Logic. It deals more distinctly and explicitly with the methodological topics treated of in Mill’s book: and the student’s grasp of the subject will be materially aided by a careful comparison of the doctrines of the two writers.

The majority of the more advanced works fall into two sections: those which are read mainly for their own historic interest or the historic information which they contain; and those which require some knowledge of mathematics or physical science, as analysing the methods, or appealing to the notation of, those sciences. In the former class Bacon’s Novum Organon claims attention from its importance in the development of English scientific speculation. The best brief introduction to it is still to be found in the essay by R. L. Ellis, in the first volume of the collected works of Bacon by him and Mr Spedding. Much valuable information and criticism is also given in Professor Fowler’s very complete edition of the Novum Organon. Ueberweg’s System of Logic is valuable to the English reader for its abundant historic references, and because it presents him with a general view of the science familiar on the continent but not readily to be gained from the ordinary English hand-books.

The student is recommended to read the logical parts of Mill’s Examination of Hamilton, less for their destructive side, in the way of criticism of Hamilton, than for the many points on which they serve to supplement Mill’s own system of Logic, and to explain the philosophic scheme which underlies that system.

Many of the advanced books on Logic which it is usual to study for the second part of the Tripos deal largely with questions pertaining to Metaphysics as described in the schedule. Among books of this class probably the Logics of Lotze and of Sigwart will furnish the best basis of study: the former is already translated and a translation of the latter is in progress. To the same class— Higher Logic it is sometimes called—belong Bradley’s Principles of Logic and Bosanquet’s Logic or Morphology of Knowledge, both of which deserve perusal.

Dr Venn’s Symbolic Logic may be taken as the best introduction to that subject and the corresponding parts of Boole’s Laws of Thought and Jevons’s Principles of Science may be studied in connexion with it. A great deal has been written on this form of Logic within the last few years and the student will find a full bibliography in Schroder’s Vorlesungen über die Algebra der Logik, Band i. 1890.

Dr Venn’s Logic of Chance may serve in like manner as an introduction to the Theory of Probabilities and the Theory of Statistics. It aims at being within the comprehension of those who have only an elementary knowledge of mathematics. Two of the best books dealing specially with statistics are Maurice Block, Traité théorique et pratique de statisque 1878, and Georg Mayr, Die Gesetzmässigheit im Gesellschaftsleben 1877.

In addition to the books already mentioned on the subject of Higher Logic and Method the two large volumes by Wundt—Logik: Erkenntnisslehre; Logik, Methodenlehre—may be consulted and will serve to introduce many other books dealing with special questions to the notice of the student.

2. Political Economy.

Of the books included in the syllabus drawn up by the Board, Mill’s Principles of Political Economy and Sidgwick’s Principles of Political Economy alone cover the whole ground as defined by the schedule for Part I. of the Moral Sciences Tripos. It will be observed, however, that only Books III. and V. of the former are recommended, and that only a portion of the latter is included in the list of works which all candidates are expected to study in detail. The reason for this, so far as Mill is concerned, is the recognition that substantial corrections are required in his general theory of Distribution. The need of such corrections was, indeed, admitted by Mill himself some time before his death; but he never faced the task of rewriting his treatise from the new point of view which he had gained. Nevertheless if the student will remember that many of the positions taken up require important modifications, he will do well to begin with a perusal of Mill’s work in order to obtain a first general survey of the subject. Professor Sidgwick’s treatise is more difficult, and should therefore be taken at a somewhat later stage.  Assuming that Mill has been read so as to gain a general idea of the ground to be covered, but without any considerable amount of attention having been paid to points of detail, the student should seek thoroughly to master Marshall’s Economics of Industry. This work should be supplemented by Walker on the Wages Question and on Land and its Rent. Here and elsewhere the differences of view between the authors read should be carefully noted and thought over. The student will find it specially useful to make a critical comparison of the theories of wages and profits laid down by Mill, Marshall, and Walker, observing both their points of resemblance and their points of difference.

The study of the general theory of Distribution and Exchange may later on be completed, so far as Part I. of the Tripos is concerned, by a careful study of Marshall’s Principles of Economics, Vol. I., and of the corresponding portions of Sidgwick’s Principles of Political Economy. Attention may be specially called to the part played by the principle of Continuity in the former work, and to the recognition by both writers of the complicated interactions between economic phenomena, which render it impossible to sum up in cut-and-dried formulas the conclusions ultimately reached.

Passing to the subject of currency and banking, the student should read Jevons’s Money and the Mechanism of Exchange and Nicholson’s Money and Monetary Problems, Part I., which usefully supplement one another. The former is mainly of a descriptive character, while the latter deals with the more difficult problems relating to the principles that regulate the value of money. Bagehot’s Lombard Street treats of the English banking system with special reference to the position of the Bank of England in the London Money Market. The above may be supplemented by Walker’s Money, Trade, and Industry, and by the corresponding chapters of Sidgwick.

The subject of international values and allied topics may be studied in Bastable’s Theory of International Trade. Goschen’s Foreign Exchanges is in some respects difficult, but it should on no account be omitted; it will give the student a fuller grasp of facts, the apprehension of which is of fundamental importance both for the theory of foreign trade and for the theory of money. Giffen’s Essays in Finance, Second Series, may be read with advantage at about this point.

Passing from economic science in the stricter sense to its applications, and considering Government interference in its economic aspects and the principles of taxation and State finance, Mill, Book V. should be supplemented by Sidgwick, Book III  A study of Professor Sidgwick’s method will afford the student a most valuable training in the philosophic treatment of practical questions.

Some of Macmillan’s English Citizen Series may here be consulted; e.g., Wilson’s National Budget, Fowle’s Poor Law, and Jevons’s State in relation to Labour. The subject of Free Trade and Protection is treated in detail, from the Free Trade standpoint, in Fawcett’s Free Trade and Protection and in Farrer’s Free Trade versus Fair Trade. Current socialistic doctrines will be found fully described and criticized in Rae’s Contemporary Socialism. The student will learn much from following the economic movements of his own time; but he must be cautioned against giving undue attention to controversial questions of the day, such as bimetallism, socialism, &c. Time may thus be occupied, which should be given to systematic study of the foundations of the science.  The scope of Political Economy, the methods employed in it, and its relations to other sciences, are treated of in Marshall’s Principles of Economics, Book I., and in Sidgwick’s Introduction. Cossa’s Guide to the Study of Political Economy and Keynes’s Scope and Method of Political Economy may also be consulted.  It would be out of place here to attempt to give detailed advice to students taking Advanced Political Economy in Part II. of the Tripos. They may be warned, however, of the importance of not neglecting to go over again more than once the ground they have already covered. They will thus familiarise themselves with the general principles of economic reasoning, and will know how to set about the solution of any new and complex problem that may be placed before them. In particular they should return again and again to the more difficult parts of Marshall and Sidgwick, and—in connexion with the former—should study the application of symbolic and diagrammatic methods to Economics. From this point of view Cournot’s Principes Mathématiques de la Théorie des Richesses and Jevons’s Theory of Political Economy should be read. Some of Jevons’s doctrines are expounded with great lucidity in Wick- steed’s Alphabet of Economic Science, and this book may be specially recommended to those students whose mathematical reading is not so far advanced as to render needless an elementary exposition of the conceptions upon which the Differential Calculus is based. A critical study of Ricardo’s Principles of Political Economy and of his Tracts on Money must not be omitted; while in order to obtain some knowledge of recent developments of theory by his latest critics—the economists of the Austrian school—reference may be made to Böhm-Bawerk’s Capital and Interest and Positive Theory of Capital, the former of which is however open to the charge of doing less than justice to the writer’s predecessors.

Every student of Economics ought to read at least some portions of the Wealth of Nations, Professor Nicholson’s edition of which, with Introduction and notes, may be recommended. Many real and fundamental divergences from modern theory will be observed, especially in Books I. and II.; but Adam Smith is generally stimulating and instructive even when the doctrines which he lays down need correction. As regards the course of economic history, especially the course and aims of economic legislation at different periods, Books III., IV., and V. are specially important. For further historical study choice may be made from the following: Ashley, Economic History; Cunningham, Growth of English Industry and Commerce; Maine, Village Communities; Seebohm, The English Village Community; Brentano, On the History and Development of Gilds; Gross, The Gild Merchant; Rogers, Six Centuries of Work and Wages; Toynbee, The Industrial Revolution; Levi, History of British Commerce. Blanqui’s History of Political Economy in Europe and Ingram’s History of Political Economy may also be read ; but it must be remembered that the latter is written from the point of view of the Comtist critic and is strongly partisan. The use of statistics in Economics may be studied in Jevons’s Investigations in Currency and Finance (edited by Professor Foxwell) and in both series of Giffen’s Essays in Finance.

A long list of useful books on various departments of Political Economy might here be added, but it must suffice specially to mention the collected Essays of J. S. Mill, Bagehot, Cairnes, and Cliffe Leslie. Portions of the following may be consulted in libraries on particular points: Eden, State of the Poor; Porter, Progress of the Nation; Tooke and Newmarch, History of Prices; Schönberg, Handbuch der politischen Oekonomie.

3. Psychology.

The Science of Psychology has made considerable advances in recent times; so that the work of earlier English writers on this subject—including even Locke—has now chiefly a historic interest. Still the student must not expect to find a perfectly clear consensus among its expositors as to its method and principles. Modern Psychology though rich in facts, is poor in definitions; and the greater part of its laws are merely empirical generalisations still awaiting further explanation.

The great difficulty in attempting to prescribe a course of reading in Psychology is to avoid repetition and what is worse—a bewildering divergence of opinion at least as regards details. There is now an English translation of Hoffding’s Outlines and with this or with Dewey’s Psychology the student had better begin. He may then read Sully’s Outlines and Bain’s works as supplementary to his first text-book. The article Psychology in the ninth edition of the Encyclopaedia Britannica is most likely to be of service to him when he feels the need of getting his psychological knowledge into more scientific form.

Psychophysics, which treats of the phenomena of mind in relation to the changes in the organism which accompany them, is a branch of Psychology to which every one who studies this subject at all, must give some attention. Here, however, we have to distinguish between the philosophical discussion of the general relation of mind and body, and a knowledge of the particular connexions between mental and corporeal phenomena. The former subject belongs rather to Metaphysics; an elementary knowledge of the latter may be gained from Prof. Ladd’s Outlines of Physiological Psychology which has just appeared and may be taken to supersede his larger Elements: it will also probably enable the student to dispense with Bernstein’s Five Senses of Man.

The advanced student of Psychology will find it a great advantage if he is able to read German. In this case Volkmann’s Lehrbuch der Psychologie will be most useful to him as a repertory of facts and opinions, besides giving the ablest exposition of the Herbartian Psychology—the Psychology which has been the most fruitful of results, at any rate in Germany. Closely related to this school is the teaching of Lotze, which should on no account be passed over: one section of his Metaphysik2 is devoted to psychological questions. His Medicinische Psychologie, long out of print and very scarce, is still worth attention: a portion of it has recently appeared in French. Drobisch’s Empirische Psychologie and Waitz’s Grundlegung, and Lehrbuch der Psychologie are works to which the student who is not pressed for time should also pay some attention. Morell’s Introduction to Mental Philosophy on the Inductive Method, is avowedly largely indebted to Waitz, Drobisch and Volkmann. It may be recommended especially to the English student who is unacquainted with German; also Ribot’s La Psychologie allemande contemporaine, which contains fair summaries of the leading doctrines of Herbart, Fechner, Lotze, Wundt and others.

2There is an English translation of this published by the Clarendon Press.

In the two large volumes of Prof. William James, Principles of Psychology, the advanced student has the means of forming an ample acquaintance with existing doctrine and current controversies. From Wundt’s Grundzüge der physiologischen Psychologie (3rd ed. 1887: French translation of the 2nd ed. 1880) the same help may be obtained as regards Experimental Psychology3. But the special knowledge required concerning the central nervous system will be got better from Dr Foster’s Text-book of Physiology, 5th ed. Parts III. and IV. There is no single book giving such facts of mental pathology as are of psychological interest. This is a department to which the French have especially devoted themselves. The following works may be mentioned :—Janet (Pierre), L’automatisme psychologique; Ribot, Les Maladies de la Mémoire; Les Maladies de la Volonté; Les Maladies de la Personnalité. Several of Ribot’s books are to be had in English.

3There is now (1891) some prospect of a Psychophysical laboratory in Cambridge. Prof. Foster has already set apart a room for the purpose and the University has made a small grant towards the purchase of apparatus. Some instruments too have been given by private donors.

Many works have recently appeared on what might be called Comparative Psychology. The subject is one that it is difficult to lift above the level of anecdote, but none the less it deserves attention. Romanes’ Mental Evolution (2 vols.) and Prof. Lloyd Morgan’s Animal Life and Intelligence will be found interesting in this department of psychology.

The origin of language and the connexion of thought and language form an important chapter of psychology and are dealt with in special works, in most of which, however, either the psychology or the philology leaves much to be desired. A general oversight of theories will be found in Marty, Ueber den Ursprung der Sprache. Max Müller’s Science of Thought, Egger’s La Parole intérieure, and Steinthal’s Einleitung in die Psychologie und Sprachwissenschaft are noteworthy.

4. Metaphysics.

The student who has already gone through a course of reading—accompanied, it is to be hoped, by oral instruction—in Psychology, will already have had his attention directed to some extent to the topics included in the schedule of Metaphysics. That this must be the case will appear, indeed, from a comparison of the two schedules of Psychology and Metaphysics respectively, independently of the books recommended. Thus it would be impossible to treat of the “data and fundamental conceptions” of Psychology, of “perception,” “intuition of things,” or “thought and abstraction,” without at the same time discussing to a certain extent the “nature and origin of knowledge” and the “relation of the individual mind to the universe,” &c.

But the principle of the separation adopted in the Cambridge scheme may perhaps be made partially clear without entering on matters of controversy; and it will probably assist the student to keep it in view from the outset. He must understand then, that Psychology deals with cognitive acts or states primarily as one class (among others) of mental phenomena; as forming part of the stream of consciousness of certain particular minds, whose processes the student is able to observe directly or indirectly. Whereas in the investigation of knowledge and its conditions that constitutes one department of Metaphysics, the same acts or states are primarily considered as representative of or related to the objects known. Or—to present substantially the same difference in another form—in investigating perceptions or thoughts from the point of view of Psychology we are no more occupied with those that are real or valid, than with those that are illusory or invalid—in fact, the latter may often be more interesting as throwing more light on the general laws of human minds: whilst as metaphysicians we are primarily concerned with real knowledge or truth as such, and treat of merely apparent knowledge or error only in order to expose and avoid it.

Under the head of Metaphysics it is intended to require a general knowledge (1) of what is coming to be called Epistemology and (2) of the speculative treatment of the fundamental questions concerning Nature and Mind prevalent at the present time, without direct reference to the History of Philosophy. Still it can scarcely be denied that the student who purposes to take up the History of Philosophy as a special subject will find some acquaintance with this history a help to the understanding of Philosophy in its most recent phases. If for no other reason this will be found true from the simple fact that nearly every writer on philosophical problems assumes some familiarity on the part of his readers with the writings of his predecessors. In particular those who are taking up both subjects and have to begin their work in private—during the Long Vacation, for instance— will find it advantageous to take up certain parts of the general history before attempting to do much at Metaphysics as outlined in the schedule, and especially to take up those parts of it that relate to the Theory of Knowledge. For these at least a general acquaintance with Hume and Kant will be helpful. Still those who are meaning to specialise in other directions can begin without this preliminary study of the history, and may reasonably count on getting what they need in this respect from lectures. Such may read some brief exposition of the Kantian philosophy, the three constructive chapters in Mill’s Examination of Hamilton (entitled Psychological Theory of Matter, Mind &c.), Mr Herbert Spencer’s First Principles and Lotze’s Metaphysics, as a preparation for lectures. Those familiar with German will find Riehl’s Philosophische Kriticismus, Kroman’s Unsere Naturerkenntniss and Wundt’s System der Philosophie useful books.

5. Politics.

The student will find all the aspects of this subject most fully dealt with in Dr Sidgwick’s Elements of Politics. This work is written from the Utilitarian point of view: the following books written from the same general standpoint may be read along with it:—Mill’s Utilitarianism, Chap, V., and Representative Government, Bentham’s Principles of Morals and Legislation, Principles of the Civil Code and Fragments on Government, and Austin’s Jurisprudence. For a treatment of the subject from a different point of view, the student may be recommended to read Green’s Lectures on Political Obligation (in the 2nd volume of his Collected Works); also Ritchie’s Principles of State Interference. Mr Herbert Spencer’s writings may also be profitably consulted, especially his Sociology, Part II. and Part V., and his volume on Justice.

The following works will be found useful for occasional reference—Bluntschli, Lehre vom modernen Staat, Vol. I. (authorised English translation published by the Clarendon Press), Maine’s Ancient Law, Early History of Institutions, and Popular Government, Stephen’s English Thought in the Eighteenth Century, Spencer’s Man versus the State, Dicey’s Law of the Constitution, Bryce’s American Commonwealth, Stirling’s Philosophy of Law, Hume’s Essays, II.—IX., and XII., Locke’s Essay on Civil Government, &c.

To those who have time and inclination to go beyond the limits of the schedule and study the history of the subject Janet’s Histoire de la Science Politique may be recommended. But some acquaintance with the original works of the more important writers is desirable—e.g., the Republic and Laws of Plato, the Ethics and Politics of Aristotle, Hobbes’s Leviathan, Montesquieu’s Esprit des Lois, Rousseau’s Contrat Social, Burke’s Thoughts on the Present Discontents and Reflections on the Revolution in France, Hegel’s Rechtsphilosophie and Philosophy of History, Comte’s Philosophie Positive, Part VI. Physique Sociale, (Vol. II. of Miss Martineau’s Translation), and Politique Positive (translated by various writers). Students ought not, however, to attempt to master the details of any of these works. On Comte, Caird’s Social Philosophy of Comte will be found useful.

6. Ethics.

Every student will naturally desire to have from the first a clear idea of the scope of the science. Unhappily there is no book from which such an idea can be gained in a quite satisfactory manner: for the degree of emphasis which is laid on different questions, and even to some extent the nature of the questions themselves, vary considerably in the different schools of ethical thought. A general sketch of the topics discussed by modern ethical writers may be found in such a book as Dewey’s Outlines of Ethics. But the significance of the various questions can hardly be fully appreciated without some reference to the history of the subject. It would be well therefore to read ch. IV. of Dr Sidgwick’s short History of Ethics at an early stage. This book is almost entirely limited, in the modern parts, to the history of English thought; but this deficiency may easily be corrected as the student proceeds with his work.

After having in this way acquired a general idea of the subject, the student may proceed to consider, more in detail, the various points of view from which the subject has been approached. He will soon find that the main schools of ethical thought group themselves naturally under the following heads:—(1) Intuitional, (2) Utilitarian, (3) Evolutionist, (4) Idealistic. As the student advances, he may be led to see that the distinction between these schools is not an absolute one, and that to a considerable extent their views overlap. But at first it may be convenient to study them separately. As representative of the Intuitional theory, the student may read the part of Martineau’s Types of Ethical Theory which contains the statement of the writer’s own doctrine— i.e. especially Part II., Book I., and perhaps the chapters on Intuitionism in Calderwood’s Handbook of Moral Philosophy; while, as representative of the Utilitarian point of view he may take Mill’s Utilitarianism, together with the criticism and further development of Mill’s ideas in Dr Sidgwick’s Methods of Ethics. The criticisms of Intuitionism in Dr Sidgwick’s Methods of Ethics and of Utilitarianism in Green’s Prolegomena to Ethics and in Sorley’s Ethics of Naturalism ought also to be studied in this connexion. With reference to Evolutionist Ethics, Mr Herbert Spencer’s Data of Ethics ought to be carefully studied, while those who have time may consult in addition such books as Mr Leslie Stephen’s Science of Ethics, Mr Alexander’s Moral Order and Progress, and Höffding’s Ethik. For criticism of the Evolutionist Ethics, reference may be made to Green’s Prolegomena to Ethics, Sidgwick’s Methods of Ethics, and Sorley’s Ethics of Naturalism. The Idealistic Ethics rests primarily on the teaching of Kant, and the best introduction to it may be found in his Metaphysic of Morals (of which Abbott’s translation is the most accurate). Dewey’s Outlines of Ethics are also written from this point of view. So are Bradley’s Ethical Studies and Green’s Prolegomena to Ethics; but only certain portions of these books can be studied with advantage by those who are not at the same time studying Metaphysics. The most complete exposition and criticism of Kant’s ethical position is to be found in the 2nd volume of Caird’s Critical Philosophy of Immanuel Kant. Among recent books Paulsen’s System der Ethik is singularly rich and suggestive.

Students who are reading Metaphysics in conjunction with Ethics will naturally bestow more attention on the fundamental difficulties of the subject than other students can be expected to give. On this, as on other aspects of Philosophy, the works of Kant will necessarily be studied with care. Green’s Prolegomena to Ethics may be strongly recommended as the most important English book dealing with the relation of Metaphysics to Ethics. Few students will find time to acquire more than a general knowledge of such speculations as those of Plato, Spinoza, and Hegel.

Students of Politics, on the other hand, may be expected to be especially interested in the relations of Ethics to the Philosophy of society and of the state. Among modern writers, the Germans have devoted most attention to this aspect of the subject, from Hegel’s Rechtsphilosophie onwards. Paulsen’s System der Ethik may be recommended; also Hoffding’s Ethik, translated from the Danish. In English, Green’s Prolegomena to Ethics and Lectures on Political Obligation (in the 2nd volume of his Collected Works) may be consulted. Several writers of the Utilitarian school have also dealt with this subject. Bentham’s Principles of Morals and Legislation and Principles of the Civil Code will be found interesting; and highly instructive discussions of various aspects of the subject are to be found in Dr Sidgwick’s Methods of Ethics, Principles of Political Economy, and Elements of Politics.

7. History of Philosophy.

A particular portion4 of the whole subject will be selected from time to time, which the student will be required to know thoroughly: and he should endeavour to avail himself of this special knowledge so as to make his general survey of the course of metaphysical speculation, in ancient or modern times, less superficial than it would otherwise be; by keeping prominently in view the connexion of the doctrines specially studied with antecedent and subsequent thought.

4The special subject selected for the examination in 1892 is, The Philosophy of Kant; and for 1893:—European Philosophy from 1600 to 1660 with special reference to Descartes, Bacon and Hobbes.

There are no good general histories of Philosophy by English writers, but there are translations of several standard histories by Germans. Of these Schwegler’s, though very brief, is good for a general survey. Erdmann is fairly full and would be excellent if not obscured in parts by careless translation. Ueberweg attempts—in the style of Prof. Bain’s Ethical Systems—to summarize in the writers’ own words but not always with Prof. Bain’s success.

The student should try, if possible, to read something of the philosophical classics at first hand. Such short works, for example, as Descartes’ Discourse on Method or his Meditations, Berkeley’s Hylas and Philonous, Hume’s Treatise of Human Nature, Vol. I., and Kant’s Prolegomena to every future metaphysic, might be read.

Prof. Sidgwick’s History of Ethics will be found the most useful text-book; and may be supplemented by Jodl’s Geschichte der Ethik. Help will also be obtained from Mr Leslie Stephen’s History of English Thought in the Eighteenth Century; Martineau’s Types of Ethical Theory; the Introduction to the second volume of Hume’s Works in the edition of Green and Grose (reprinted in the first volume of Green’s Collected Works); and Wundt’s Ethik, Abschnitt II.

General Remarks on Method and Time of Study.

1. Method of reading.

Perhaps the best plan upon commencing a new work is to read it rapidly through first, in order to form a general notion of its bearing and to catch its principal points. The first reading may be too careful. The student may find himself face to face with difficulties, which, although really only of an incidental character, may cause him to misconceive the proportions of the whole, if he have formed a determination—in itself praiseworthy —to master every part upon first acquaintance. Upon the second reading, an analysis should be made of the more important works, but care should be taken that it do not become long and wearisome: it should be distinctly of the nature of a summary, and not a mere series of extracts. Such analyses are almost indispensable, to enable the student to perform, in the concluding period of his course, an effective and systematic revision of the whole results of his study. Further, at the second time of reading, the student should take careful note of any difficulties that he may find in understanding the doctrines or criticisms propounded, or any doubts that may occur to him as to their correctness. He need not be afraid of losing time by writing down in his note-book as precise a statement as possible of his doubt or difficulty; since no exercise of his mind is likely to be more conducive to his attaining a real grasp of his subject. He will sometimes find that the mere effort to state a difficulty clearly has the effect of dispelling it; or, if not at the time, at any rate when he recurs to the point on a subsequent day he will often find the problem quite easy of solution: while in the cases where his perplexity or objection persists, a clear statement of it will generally bring his mind into the most favourable condition for receiving explanations from his teacher.

In subjects so full of unsettled controversy as the Moral Sciences generally are, a student must be prepared to find himself not unfrequently in legitimate disagreement with the authors studied; (though he should not hastily conclude that this is the case, especially during the earlier stages of his course). In all except quite recent books, he is likely to find some statements of fact or doctrine which all competent thinkers at the present day would regard as needing correction; while in other cases he will find, on comparing different works, important discrepancies and mutual contradictions on points still debated between existing schools of thought. He should carefully note the results of such comparisons; but he should not content himself with merely committing them to memory; rather, he should always set himself to consider from what source each controversy arises, what its relation is to the rest of the doctrine taught in the works compared, and by what method the point at issue is to be settled.

It will generally be found convenient to put in tabular form any divisions or classifications which , are met with in the books read. Such lists are not indeed necessarily of great importance in themselves, but they furnish a convenient framework for criticisms and comparisons of the methods and results of various writers.

The constant practice of writing answers to papers of questions and longer compositions on special points arising out of the subjects studied, cannot be too strongly urged. Many minds are hardly able to bring their grasp of subtle or complicated reasonings to the due degree of exactness and completeness, until their deficiencies in these respects have been brought home to them by exercises in written exposition.

2. Time of study.

A student who is in a position to begin effective work in his first term may hope to be prepared for Part I. of the Tripos in his second year, and may take Part II. at the end of his third, assuming, of course, in both cases that he does a reasonable amount of private study during Long Vacations. But it is desirable, when circumstances admit of it and especially if two of the special subjects are taken up, to devote not less than two years to the work of the Second Part.

Those who have taken honours in other Triposes at the end of their second year, will be able afterwards to prepare fully for either part of the Moral Sciences Tripos at the end of their fourth year, without being inconveniently pressed for time—supposing them to read steadily in their second, as well as in their third Long Vacation. If, however, the period entirely devoted to this preparation is only one year—as must be the case with students who take some other Tripos at the end of their third year—it is very desirable that some part of the subjects should have been read at an earlier stage of the course.

The Special Board for Moral Science publishes annually, towards the end of the Easter Term, a list of lectures for the coming academical year in different departments of the Moral Sciences. These lectures are, generally speaking, so arranged as to provide all the oral instruction required by students at different stages of their course.

Source:  Dr. J. Ward, Trinity College, editor: Part VIII. The Moral Sciences Tripos  in The Student’s Guide to the University of Cambridge (5th edition, rewritten). Cambridge (U.K.): Deighton, Bell and Co., 1891.

 

Image Source:  Illustration by Edward Hull “The New Court, Trinity College Cambridge” from page 81 of  Alfred J. Church, The Laureate’s Country. London: Seeley, 1891.